Health Assessment Final

Réussis tes devoirs et examens dès maintenant avec Quizwiz!

What is the most appropriate recommendation to prevent cervical cancer in females? A. Administration of HPV vaccine around 11 and 12 years old B. Start birth control if sexually active C. There is nothing that can prevent cervical cancer. D. Use barrier protection method during sexual intercourse.

A

What type of pain is short and self-limiting and dissipates after the injury heals? A. Acute B. Persistent C. Chronic D. Breakthrough

A

When assessing mental status in children, what statement is true? A. All aspects of mental status in children are interrelated B. Children are highly labile and unstable until the age of 2 C. Children's mental status is largely a function of their parents' level of functioning until the age of 7 D. A child's mental status is impossible to assess until the child develops the ability to concentrate

A

When assessing the tongue, you should: A. Palpate the U-shaped area under the tongue B. Check tongue color for cyanosis C. Use a tongue blade to elevate the tongue while placing a finger under the jaw D. Ask the person to say "ahhh" and note a rise in the midline

A

When auscultating the heart, your first step is to: A. Identify S1 and S2 B. Listen for S3 and S4 C. Listen for murmurs D. Identify all 4 sound on the first round

A

When inspecting the eyeballs of an African American individual, which of the following might the examiner expect to observe? A. Small brown macules on the sclera B. A slight amount of drainage around the lacrimal apparatus C. A slight yellow discoloration of the sclera D. A slight misalignment of the eyeballs

A

When performing a health history, the nurse would note immunizations under which category? A. Past medical history B. Family history C. History of present illness D. Personal history

A

When assessing the carotid artery, the nurse should palpate: A. Bilaterally at the same time while standing behind the patient B. Medial to the sternomastoid muscle, one side at a time C. For a bruit while asking the patient to hold his or her breath briefly D. For unilateral distention while turning the patient's head to one side

B

When completing a health assessment, which of the following actions most demonstrates cultural competence? A. Ask about family history of diseases. B. Ask about use of traditional, herbal, or folk remedies. C. Make sure the blood pressure cuff fits appropriately. D. Measure height and weight in a private room.

B

Which of the following represent the appropriate vital signs and sequence for obtaining vital signs on a healthy six-week-old baby? A. Rectal temperature, respirations, pulse rate B. Respirations, apical pulse rate, axillary temperature C. Respirations, pulse rate, blood pressure, rectal temperature D. Oral temperature, respirations, pulse rate, blood pressure

B

Which of the following respiratory rates recorded for an infant without chronic illness would require further interventions and assessment by the nurse? A. 27 B. 45 C. 35 D. 30

B

During an assessment, the nurse knows that expected assessment findings in the normal adult lung include the presence of: A. adventitious sounds and limited chest expansion. B. increased tactile fremitus and dull percussion tones. C. muffled voice sounds and symmetrical tactile fremitus. D. absent voice sounds and hyperresonant percussion tones.

C

During examination of the scrotum, a normal finding would be that: A. The left testicle is firmer to palpation than the right B. The left testicle is larger than the right C. The left testicle hangs lower than the right D. The left testicle is more tender to palpation than the right

C

During the evaluation phase of the nursing process, which action would be included? A. Validating the nursing diagnosis B. Establishing priorities related to patient care C. Including the patient and family members D. Establishing a timeline for planned outcomes

C

During the examination of a 70-year-old man, you note gynecomastia. You would: A. Refer for a biopsy B. Refer for a mammogram C. Review the medications for drugs that have gynecomastia as a side effect D. Proceed with the examination. This is a normal part of the aging process

C

During the examination of an infant, use the cotton-tipped applicator to stimulate the anal sphincter. The absence of a response suggests a lesion of: A. L2 B. T12 C. S2 D. C5

C

During the examination of the genitalia of a 70-year-old woman, a normal finding would be: A. Hypertrophy of the mons pubis B. Increase in vaginal secretions C. Thin and sparse pubic hair D. Bladder prolapse

C

During the physical examination, your patient is diaphoretic and pale and complains of dull pain in the LUQ of the abdomen. This is what type of pain? A. Cutaneous pain B. Somatic pain C. Visceral pain D. Psychogenic pain

C

Endogenous obesity is A. characterized by evenly distributed excess body fat. B. due to inadequate secretion of cortisol by the adrenal glands. C. caused by excess adrenocorticotropin (ACTH) production by the pituitary gland. D. a result of excessive secretion of growth hormone in adulthood.

C

For a woman, history of her mother's health during pregnancy is important. A medication that requires frequent follow-up is: A. Corticosteroid B. Theophylline C. Diethylstilbestrol D. Aminoglycoside

C

Hematuria is a term used for A. bleeding after intercourse. B. bloody discharge. C. blood in the urine. D. urine in the blood.

C

Identify the blood vessel that runs diagonally across the sternomastoid muscle. A. Temporal artery B. Carotid artery C. External jugular vein D. Internal jugular vein

C

If the tympanic membrane has white dense areas, the examiner suspects: A. a fungal infection B. serous fluid from serous otitis media C. scarring from recurrent ear infections D. perforation from a ruptured membrane

C

In examining a 70-year-old male patient, the nurse notices that he has bilateral gynecomastia. Which of the following describes the nurse's best course of action? A. Recommend that he make an appointment with his physician for a mammogram. B. Ignore it. Benign breast enlargement in men is not unusual. C. Explain that this condition may be the result of hormonal changes, and recommend that he see his physician. D. Explain that gynecomastia in men is usually associated with prostate enlargement and recommend that he be thoroughly screened.

C

In examining the ear of an adult, the canal is straightened by pulling the auricle: A. Down and forward B. Down and back C. Up and back D. Up and forward

C

In pulsus paradoxus A. the rhythm is regular, but the force of the pulse varies with alternating beats. B. there is a deficiency of arterial blood to a body part. C. beats have weaker amplitude with respiratory inspiration and stronger amplitude with expiration. D. the rhythm is irregular; every other beat is premature.

C

In what section of the health assessment record would the nurse document, "reports usual health "ok" with no recent weight changes"? A. Functional assessment B. Physical examination—General Survey C. Review of Systems—General Health D. Past medical history

C

In young children, the thymus gland A. is small and begins to atrophy. B. produces B lymphocytes. C. produces T lymphocytes. D. is not important in immune function.

C

Inspection of a person's right hand reveals a red, swollen area. To further assess for infection, you would palpate the: A. cervical node B. axillary node C. epitrochlear node D. inguinal node

C

Intermittent claudication includes: A. Muscular pain relieved by exercise B. Neurologic pain relieved by exercise C. Muscular pain brought on by exercise D. Neurologic pain brought on by exercise

C

Mean arterial pressure is: A. The arithmetic average of systolic and diastolic pressures B. The driving force of blood during systole C. Diastolic pressure plus one third of the pulse pressure D. Corresponding to phase II Korotkoff

C

Mental status assessment documents A. schizophrenia and other mental health disorders. B. artistic or writing ability in the mentally ill person. C. emotional and cognitive functioning. D. intelligence and educational level.

C

Most facial bones articulate at a suture. Which facial bone articulates at a joint? A. Zygomatic bone B. Nasal bone C. Mandible D. Maxilla

C

Mr. B tells you, "Everyone here ignores me." You say, "Ignores you?" This technique is best described as A. Clarification B. Selective language C. Reflecting D. Validation

C

To test for stereognosis, you would: A. Have the person close his or her eyes and then raise the person's arm and ask the person to describe its location B. Touch the person with a tuning fork C. Place a coin in the person's hand and ask him or her to identify it D. Touch the person with a cold object

C

Using the otoscope, the tympanic membrane is visualized. The color of the membrane is: A. Deep pink B. Creamy white C. Pearly gray D. Dependent on the ethnicity of the individual

C

Vaginal lubrication during intercourse is produced by A. sebaceous glands. B. adrenal glands. C. Bartholin glands. D. Skene glands.

C

What can be determined when the nurse clusters data as part of the critical-thinking process? A. This step identifies problems that may be urgent and require immediate action B. This step involves making assumptions in the data C. The nurse recognizes relevant information among the data D. Risk factors can be determines to the nurse knows how to offer health teaching

C

What information is included in greater detail when taking a health history on an infant? A. Environmental hazards B. History of present illness C. Nutritional data D. Family history

C

What is the cause of the red reflex? A. Petechial hemorrhages in the sclera B. Diabetic retinopathy C. Light reflecting from the retina D. Blood in the vitreous

C

What is the most common site of nose bleeds? A. The turbinates B. The columellae C. Kiesselbach plexus D. The meatus

C

Which would be considered a risk diagnosis? A. Identifying existing levels of wellness B. Evaluating previous problems and goals C. Identifying potential problems the individual may develop D. Focusing on strengths and reflecting an individual's transition to higher levels of wellness

C

Which would be included in a holistic model of assessment? A. Nursing goals for the patient B. Anticipated growth and development patterns C. A patient's perception of his or her health status D. The nurse's perception of disease related to the patient

C

Craniosynostosis is a severe deformity caused by: A. Increased intracranial pressure B. Excess growth hormone or a deficit in thyroid hormone C. A localized bone disease that softens, thickens, and deforms bone D. Premature closure of the sutures

D

Darwin tubercle indicates: A. An overgrowth of scar tissue B. A blocked sebaceous gland C. A sign of gout called tophi D. A congenital, painless nodule at the helix

D

Deep palpation is used to A. elicit deep tendon reflexes. B. evaluate surface characteristics. C. determine the density of a structure. D. identify abdominal contents.

D

During an interview, the nurse states, "You mentioned having shortness of breath. Tell me more about that." Which verbal skill is used with this statement? A. Reflection B. Facilitation C. Direct question D. Open-ended question

D

During assessment of extraocular movements, two back-and-forth oscillations of the eyes in the extreme lateral gaze occur. This response indicates: A. that the patient needs to be referred for a more complete eye examination. B. this assessment should be repeated in 15 minutes to allow the eyes to rest. C. a disease of the vestibular system, further evaluation is needed. D. an expected movement of the eyes during this procedure.

D

During palpation of the testes, the normal finding would be: A. Firm to hard and rough B. Nodular C. 2 to 3 cm long x 2 cm wide and firm D. Firm, rubbery, and smooth

D

During the assessment of the spine, the patient would be asked to: A. Adduct and extend B. Supinate, evert, and retract C. Extend, adduct, invert, and rotate D. Flex, extend, abduct, and rotate

D

During the assessment, which part of the hand is best for detecting vibration? A. Fingertips B. Index finger and thumb in opposition C. Dorsum of the hand D. Ulnar surface of the hand

D

During the examination of the breasts of a pregnant woman, you would expect to find: A. Peau d'orange B. Nipple retraction C. Unilateral, obvious venous patter D. Blue vascular pattern over both breasts

D

During transillumination of a scrotum, you note a nontender mass that transilluminates with a red glow. This finding is suggestive of: A. scrotal hernia B. scrotal edema C. orchitis D. hydrocele

D

Fibrous bands running directly from one bone to another that strengthen the joint and help prevent movement in undesirable directions are called: A. Bursa. B. Tendons. C. Cartilage. D. Ligaments.

D

Flattening of the angle between the nail and its base is: A. Found in subacute bacterial endocarditis B. A description of spoon-shaped nails C. Related to calcium deficiency D. Described as clubbing

D

For older adult postoperative patients, poorly controlled acute pain places them at higher risk for: A. Atelectasis B. Increased myocardial oxygen demand C. Impaired wound healing D. All of the above

D

For what or with whom should physical touch be used during the interview? A. Only with individuals from a Western culture B. As a routine way of establishing contact with the person and communication empathy C. Only with patients of the same gender D. Only if the interviewer knows the person well.

D

Functional assessment measures how a person manages day-to-day activities. The impact of adoption on the daily activities of a child is referred to as... A. Development history B. Instrumental activities of daily living C. Reason for seeking care D. Interpersonal relationship assessment

D

Gynecomastia occurs with A. calcium channel blockers. B. Addison disease. C. hypothyroidism. D. liver cirrhosis.

D

H.T. has come to the clinic for a follow-up visit. Six months ago, he was started on new medication that may cause erectile dysfunction as a side effect; therefore medication classes explored by the nurse are: A. Antipyretics B. Bronchodilators C. Corticosteroids D. Antihypertensives

D

Hematopoiesis takes place in which of the following? A. Liver B. Spleen C. Kidneys D. Bone marrow

D

Hyperactive bowel sounds are: A. High-pitched B. Rushing C. Tinkling D. All of the above

D

If the thyroid gland were enlarged bilaterally, which maneuver would be appropriate for you to assess? A. Check for deviation of the trachea B. Listen for a bruit over the carotid arteries C. Listen for a murmur over the aortic area D. Listen for a bruit over the thyroid lobes

D

In addition to initiating digestion of food, saliva: A. protects the mucosa from caustic substances B. augments taste sensation C. inhibits overgrowth of bacteria in the mouth D. cleans and protects the mucosa

D

In recording the childhood illnesses of a patient who denies having had any, which note by the nurse would be most accurate? A. Patient denies usual childhood illnesses. B. Patient states he was a "very healthy" child. C. Patient states his sister had measles, but he didn't. D. Patient denies measles, mumps, rubella, chickenpox, pertussis, and strep throat.

D

Increased tactile fremitus would be evident in an individual who has which of the following conditions? A. Crepitus B. Emphysema C. Pneumothorax D. Pneumonia

D

Loss of the mucus plug is a A. presumptive sign of pregnancy. B. probable sign of pregnancy. C. positive sign of pregnancy. D. sign of impending labor.

D

Older adults have: A. increased gastric acid secretion B. increased liver size C. decreased incidence of gallstones D. abdominal musculature less tone

D

On auscultating a patient, you not a coarse, low-pitched sound during both inspiration and expiration. This patient reports pain with breathing. These findings are consistent with: A. Fine crackles B. Wheezes C. Atelectasis crackles D. Pleural friction rub

D

On first encounter with a client, the nurse observes the client walking into the room and seating himself in a chair. The purpose of the is observation includes which of the following? A. It provides data regarding the client's attitude B. It provides data to include in the subjective portion of the assessment C. It provides data to confirm a diagnosis D. It provides data regarding strength and symmetry of parts of the body

D

On palpation the prostate gland is enlarged, nontender, firm, and smooth with a palpable central groove. This assessment finding indicates: A. a normal prostate gland B. prostatitis C. prostate carcinoma D. benign prostatic hypertrophy

D

Oral malignancies are most likely to develop: A. On the soft palate B. On the tongue C. In the buccal cheek mucosa D. In the mucosal "gutter" under the tongue

D

Pain in the aging adult is considered to be: A. Part of the normal degenerative process B. Perceived to a lesser degree C. An expected finding D. Unrelated to the aging process

D

Pain signals are carried to the central nervous system by way of A. perception. B. modulation. C. referred pain. D. afferent fibers.

D

Percussion of the chest is A. is not influenced by the overlying chest muscle and fat tissue. B. normal if a dull note is elicited. C. a useful technique for identifying small lesions in lung tissue. D. helpful only in identifying surface alterations of lung tissue.

D

Physical appearance includes statements that compare appearance with A. mood and affect. B. nutrition. C. gait. D. stated age.

D

Providing resistance while the patient shrugs his or her shoulders is a test of which cranial nerve? A. II B. V C. IX D. XI

D

Select the best description of an accurate assessment of a patients' respirations. A. Count for a full minute before taking the pulse B. Count for 15 seconds and multiply by 4 C. Count after informing the patient where you are in the assessment process D. Count for 30 seconds after pulse assessment

D

Spirituality is defined as A. a social group that claims to possess variable traits B. participating in religious services on a regular basis C. the process of being raised within a culture D. a personal effort to find meaning and purpose in life

D

Stretch marks occurring during pregnancy are known as A. palmar erythema. B. chloasma. C. linea nigra. D. striae.

D

Striae are caused by A. elevated blood pressure during pregnancy. B. decreased fetal movement. C. hyperinsulinemia. D. rapid growth of the underlying tissue or organ.

D

Subjective data includes A. A client's laboratory reports B. Results of the physical examination C. Results of radiographic studies D. Symptoms or otherwise hidden data perceived by the client

D

Tests of fetal well-being include A. Leopold maneuvers and pelvimetry. B. HIV screening and a PPD test. C. complete blood count and a urinalysis. D. fetal heart tones and movement counts.

D

The 4 areas to consider during the general survey include: A. Ethnicity, gender, age, and socioeconomic status B. Physical appearance, gender, ethnicity, and affect C. Dress, affect, nonverbal behavior, and mobility D. Physical appearance, body structure, mobility and behavior

D

The American Academy of Pediatrics recommends newborn male circumcision because of which of the following benefits? A. Increased risk for HIV B. Increased risk for sexually transmitted infections C. Transmission of trichomoniasis D. Decreased risk for urinary tract infections

D

Which of the following would be a normal sensitivity to pressure for the testes? A. Somewhat B. Not at all C. Left more sensitive than right D. Only when inflammation is present

A

A bruit of the temporal artery is suspected when the nurse hears A. a clicking sound B. a soft blowing sound C. a should like hair rustling D. a vibration

B

Select the correct description of the left lung. A. Narrower than the right lung with three lobes B. Narrower than the right lung with two lobes C. Wider than the right lung with two lobes D. Shorter than the right lung with three lobes

B

Cerebellar function is assessed by which of the following: A. Muscle size and strength assessment B. Cranial nerve examination C. Coordination - hopping on one foot D. Spinothalamic test

C

The tonsils are graded a 3+. The tonsils would be: A. Visible B. Halfway between the tonsillar pillars and uvula C. Touching the uvula D. Touching each other

C

A patient states that he has frothy, foul-smelling stool that float on the surface of the water in the toilet bowl. What type of stool is this patient descrbing? A. Steatorrhea B. Melena C. Dyschezia D. A parasitic infection

A

To determine if a dark-skinned patient is pale, the nurse should assess the color of the A. skin in the antecubital space. B. palms of the hands. C. conjunctivae. D. ear lobes.

C

A patient's vision is recorded as 20/80 in each eye. The nurse interprets this finding to mean that the patient: A. Has poor vision. B. Has acute vision. C. Has normal vision. D. Is presbyopic.

A

Senile tumors may resemble parkinsonism, except that senile tremors do not include: A. Nodding of the head as if responding yes or no B. Rigidity and weakness of voluntary movement C. Tremor of the hands D. Tongue protrusion

B

Signs of pregnancy identified by the clinician are considered A. signs of impending labor. B. probable signs. C. presumptive signs. D. positive signs.

B

Stridor is a high-pitched, inspiratory crowing sound commonly associated with A. pneumothorax. B. upper airway obstruction. C. congestive heart failure. D. atelectasis.

B

A positive Babinski sign is: A. Dorsiflexion of the big toe and fanning of all toes B. Plantar flexion of the big toe with a fanning of all toes C. The expected response in healthy adults D. Withdrawal of the stimulated extremity from the stimulus

A

A retention cyst in the epididymis filled with milky fluid containing sperm is called A. a spermatocele. B. a prepuce. C. a varicocele. D. Peyronie disease.

A

The "review of systems" in the health history is A. a short statement of general health status. B. an evaluation of past and present health state of each body system. C. a documentation of the problem as perceived by the patient. D. a record of objective findings.

B

Clonus that may be seen when testing deep tendon reflexes is characterized by a(n): A. set of rapid, rhythmic contractions of the same muscle. B. contraction of the muscle that appears after the tendon is hit the second time. C. additional contraction of the muscle that is of greater intensity than the first contraction. D. parallel response in the opposite extremity.

A

The CAGE test is a screening questionnaire that helps to identify A. depression. B. excessive or uncontrollable drinking. C. unhealthy lifestyle behaviors. D. personal response to stress.

B

Breast development in an adolescent girl usually A. begins between 12 and 13 years of age. B. occurs after the beginning of menstruation. C. takes an average of 5 years. D. precedes menstruation by about 2 years.

D

The Doppler technique A. is used to assess the apical pulse. B. amplifies Korotkoff sounds during blood pressure measurement. C. measures arterial oxygenation saturation. D. provides an easy and accurate measurement of the diastolic pressure.

B

Crepitation is an audible sound that is produced by: A. roughened articular surfaces moving over each other. B. flexion and extension of an inflamed bursa. C. tendons or ligaments that slip over bones during motion. D. joints that are stretched when placed in hyperflexion or hyperextension.

A

Decreased vision in an older patient may be due to which of the following conditions? A. Macular degeneration B. Presbyopia C. Fixation D. Retinoblastoma

A

During a routine visit, M.B., age 78, asks about small, round, flat, brown macules on the hands. What is your best response after assessing the areas? A. "These are the result of sun exposure and do not require treatment." B. "These are related to exposure to the sun. They may become cancerous." C. "Theses are skin tags that occur with aging. No treatment is required." D. "I'm glad you brought this to my attention. I will arrange for a biopsy."

A

CRIES is appropriate pain assessment tool for: A. Cognitively impaired older adults B. Children ages 2 to 8 years C. Infants and children D. Preterm and term neonates

D

Chadwick sign is: A. Softening of the cervix B. Rotation of the cervix to the left C. The fundus of the uterus tips forward D. A bluish color of the cervix and vaginal walls during early pregnancy

D

Palpable inguinal lymph nodes are A. normal if small (less than 1 cm), movable, and nontender. B. abnormal and indicate the presence of malignant disease. C. abnormal in adults but common in children and infants. D. normal if fixed and tender.

A

Checking for skin temperature is best accomplished by using: A. The palmar surface of the hands B. The ventral surface of the hands C. The fingertips D. The dorsal surface of the hands

D

Which sinus can you assess through examination? A. Ethmoid & sphenoid B. Frontal & ethmoid C. Maxillary & sphenoid D. Frontal & maxillary

D

Palpation of the adult client's neck reveals nonpalpable lymph nodes. This is A. A normal finding in adults B. Reason for referral to an ear, nose, and throat specialist C. Probably caused by an infection D. Cause to inspect for further malformations

A

Presumptive signs of pregnancy include A. amenorrhea and fatigue. B. fetal heart tones and cardiac activity on an ultrasound. C. enlarged uterus and fetal movement. D. brisk deep tendon reflexes and clonus.

A

Pruritus is the presence of: A. an itching or burning sensation B. a longitudinal tear in the superficial mucosa at the anal margin C. blood in the stool D. excessive fat in the stool

A

Pulse oximetry measures: A. Arterial oxygen saturation of hemoglobin B. Venous oxygen saturation of hemoglobin C. Combined saturation of arterial and venous blood D. Carboxyhemoglobin levels

A

Match the skin layer with the descriptor: Skin Layers: Epidermis, Dermis, Subcutaneous Layer 1. Basal cell layer 2. Aids protection by cushioning 3. Collagen 4. Adipose tissue 5. Uniformly thin 6. Stratum corneum 7. Elastic tissue

1. Epidermis 2. Subcutaneous layer 3. Dermis 4. Subcutaneous layer 5. Epidermis 6. Epidermis 7. Dermis

Match the term with the descriptor: Terms: Pallor, Erythema, Cyanosis, Jaundice 1. Intense redness of the skin due to excess blood in the dilated superficial capillaries 2. Bluish mottled color that signifies decreased perfusion 3. Absence of red-pink tones from the oxygenated hemoglobin in blood 4. Increase in bilirubin in the blood causing a yellow color in the skin

1. Erythema 2. Cyanosis 3. Pallor 4. Jaundice

Pulse pressure is described as: A. The difference between the systolic and diastolic pressure B. A reflection of the viscosity of the blood C. Another way to express the systolic pressure D. A measure of vasoconstriction

A

Pyrosis is: A. a burning sensation in the upper abdomen. B. an abnormally sunken abdominal wall. C. an inflammation of the peritoneum. D. a congenital narrowing of the pyloric sphincter.

A

A newborn infant has a dark green stool 2 days after brith. How should you interpret this? A. This type of stool would indicate anal patency B. The dark green color could indicate occult blood in the stool C. Meconium stool can be reflective of distress in the newborn D. The newborn should have passes the first stool within 12 hours after birth

A

A normal common breast variation is A. a supernumerary nipple. B. enlarged axillary lymph nodes. C. fixation of the breast. D. a supernumerary breast.

A

A nurse notes in a client pallor and cyanosis of the oral cavity and lips. Based on the findings the nurse would identify the client's need for A. Oxygen B. Blood transfusion C. Skin care to prevent chafing D. Fluid resuscitation

A

Match Configurations of the Thorax with Description: Configurations: Normal Chest, Barrel Chest, Pectus Excavatum, Pectus Carinatum, Scoliosis, Kyphosis 1. Anteroposterior = transverse diamete 2. Exaggerated posterior curvature of thoracic spine 3. Lateral S-shaped curvature of the thoracic and lumbar spines 4. Sunken sternum and adjacent cartilages 5. Elliptic shape with an anteroposterior to transverse diameter in the ratio of 1:2 6. Forward protrusion of the sternum with ribs sloping back at either side

1. Barrel Chest 2. Kyphosis 3. Scoliosis 4. Pectus Excavatum 5. Normal Chest 6. Pectus Carinatum

Match Lung Border with Location: Lung Borders: Apex, Base, Lateral Left, Lateral Right, Posterior Apex 1. Rests on the diaphragm 2. C7 3. Sixth rib, midclavicular line 4. Fifth intercostal 5. 3 to 4 cm above the inner third of the clavicles

1. Base 2. Posterior Apex 3. Lateral Left 4. Lateral Right 5. Apex

Right upper quadrant tenderness may indicate pathology in the: A. Liver, pancreas, or ascending colon B. Liver and stomach C. Sigmoid colon, spleen, or rectum D. Appendix or ileocecal valve

A

Match the term with the descriptor: Terms: Harlequin, Erythema toxicum Acrocyanosis, Physiologic Jaundice, Carotenemia, Cafe Au Lait Spot, Cutis Marmorata 1. Tiny, punctate red macules and papules on the cheeks, trunk, chest, back and buttocks 2. Lower half of the body turns red, upper half blanches 3. Transient mottling on trunk and extremities 4. Bluish color around the lips, hands, fingernails, feet, and toenails 5. Large round or oval patch of light brown usually present at birth 6. Yellowing of skin, sclera, and mucous membranes due to increased numbers of RBCs hemolyzed after birth 7. Yellow-orange color in light-skinned persons from large amounts of foods containing carotene

1. Erythema toxicum 2. Harlequin 3. Cutis Marmorata 4. Acrocyanosis 5. Cafe Au Lait Spot 6. Physiologic Jaundice 7. Carotenemia

Match the term to the definition: Depression, Anxiety, Flat Affect, Euphoria, Lability, Rage, Irritability, Fear, Depersonalization 1. Lack of emotional response 2. Loss of identity 3. Excessive well-being 4. Apprehensive from the anticipation of a danger whose source is unknown 5. Annoyed, easily provoked 6. Loss of control 7. Sad, gloomy, dejected 8. Rapid shift of emotions 9. Worried about known external danger

1. Flat Affect 2. Depersonalization 3. Euphoria 4. Anxiety 5. Irritability 6. Rage 7. Depression 8. Lability 9. Fear

Match the term with the definition. Terms: Pericardial Fluid, Ventricle, Endocardium, Myocardium, Pericardium, Atrium 1. Tough, fibrous, double-walled sac that surrounds and protect the heart 2. Thin layer of endothelial tissue that lines the inner surface of the heart chambers and valves 3. Reservoir for holding blood 4. Ensures smooth, friction-free movement of the heart muscle 5. Muscular pumping chamber 6. Muscular wall of the heart

1. Pericardium 2. Endocardium 3. Atrium 4. Pericardial Fluid 5. Ventricle 6. Myocardium

Rome III criteria for constipation includes which of the following? A. Straining to have a bowel movement B. Pain with bowel movement C. Soft, firm stools D. Four stools per week

A

A 40-year-old male patient has cellulitis of the left lower extremity and no history of health problems. Which of the following findings would need further evaluation? A. Irregular pulse B. 2+ pedal pulses bilaterally C. Receding hairline D. Calluses on both hands

A

A 65-year-old man has noticed a change in his personality and his ability to understand. He also cries and becomes angry very easily. The cerebral lobe responsible for these behaviors is the ______ lobe. A. Frontal B. Parietal C. Occipital D. Temporal

A

A common cause of a conductive hearing loss is: A. Impacted cerumen B. Acute rheumatic fever C. A CVA D. Otitis externa

A

A common error in blood pressure measurement is A. waiting less than 1 to 2 minutes before repeating the blood pressure reading on the same arm. B. taking the blood pressure in an arm that is at the level of the heart. C. deflating the cuff about 2 mm Hg per heartbeat. D. using a blood pressure cuff whose bladder length is 80% of the arm circumference.

A

A function of the venous system includes: A. Holding more blood when blood volume increases B. Conserving fluid and plasma proteins that leak out of the capillaries C. Forming a major part of the immune system that defends the body against disease D. Absorbing lipids from the intestinal tract

A

A known risk factor for venous ulcer development is: A. Obesity B. Male gender C. History of hypertension D. Daily aspirin therapy

A

A major characteristic of dementia is: A. Impairment of short- and long-term memory B. Hallucinations C. Sudden onset of symptoms D. Substance-induced

A

A nasal polyp is distinguished from the nasal turbinates by 3 of the following. Which reason is FALSE? A. The polyp is highly vascular B. The polyp is movable C. The polyp is pale gray in color D. The polyp is nontender

A

A patient has a barrel-shaped chest, characterized by: A. Equal anteroposterior transverse diameter and ribs being horizontal B. Anteroposterior transverse diameter of 1:2 and an elliptic shapte C. Anteroposterior transverse diameter of 2:1 and ribs being elevated D. Anteroposterior transverse diameter of 3:7 and ribs sloping back

A

A woman is in the clinic for an annual gynecologic examination. The nurse should plan to begin the interview with the: A. Menstrual history, because it is generally nonthreatening. B. Obstetric history, because it includes the most important information. C. Urinary system history, because problems may develop in this area as well. D. Sexual history, because discussing it first will build rapport.

A

A young woman has come for her first gynecologic examination. Because she has not had any children, the examiner expects the cervical os to appear: A. Smooth and circular B. Irregular and slitlike C. Irregular and circular D. Smooth and enlarger

A

Amplitude is A. the intensity (soft or loud) of sound. B. the number of vibrations per second. C. the length of time the note lingers. D. the subjective difference in a sound's distinctive overtones.

A

An adult with a body mass index (BMI) less than 18.5 kg/m2 is considered which of the following? A. Underweight B. Obesity C. Normal weight D. Overweight

A

An assessment of a 47-year-old diabetic patient reveals decreased ability to distinguish between sharp and dull sensation. The nurse further assesses for motor loos, diminished dorsalis pedis, or posterior tibial pulses, and coolness of the extremity. Which of the following best explains the rationale for including these in the assessment? A. Diabetic patients frequently have arterial insufficiency. B. Motor loss is most consistent with venous insufficiency. C. Damage with the central nervous system (CNS) and the ability to interpret sensation is frequently a problem with diabetes. D. Insulin use causes damage to microvasculature.

A

An example of a primary lesion is a(n) A. urticaria. B. erosion. C. port-wine stain. D. ulcer.

A

An example of an open-ended question or statement is A. "Tell me about your pain." B. "You are upset about the level of pain, right?" C. "On a scale of 1 to 10, how would you rate your pain?" D. "I can see that you are quite uncomfortable."

A

An older adult A. has a slower response time. B. has diminished recent and remote memory recall. C. experiences a 10-point decrease in intelligence. D. has difficulty with problem solving.

A

An older adult verbalizes to the nurse that he/she is having pain in his/her left wrist. Which of the following would be the most appropriate response by the nurse? A. "Tell me more about the pain." B. "Occasional pain is a normal part of getting older." C. "It's nothing to worry about." D. "We try to avoid prescribing pain medicine to older adults because they often become addicted."

A

Arteriosclerosis refers to A. thickening and loss of elasticity of the arterial walls. B. a sac formed by dilation in the arterial wall. C. deposition of fatty plaques along the intima of the arteries. D. a variation from the heart's normal rhythm.

A

Ascites is defined as: A. an abnormal accumulation of serous fluid within the peritoneal cavity. B. a bowel obstruction. C. an abnormal enlargement of the spleen. D. a proximal loop of the large intestine.

A

Assessment of self-esteem and self-concept is part of the functional assessment. Areas covered under self-esteem and self-concept include A. education, financial status, and value-belief system. B. family role, interpersonal relations, social support, and time spent alone. C. stressors, coping mechanisms, and change in past year. D. exercise and activity, leisure activities, and level of independence.

A

Atrophic skin changes that occur with peripheral arterial insufficiency include: A. Thin, shiny skin with loss of hair B. Brown discoloration C. Thick, leathery skin D. Slow-healing blisters on the skin

A

Auscultating the abdomen is begun in the right lower quadrant (RLQ) because: A. Bowel sounds are always normally present here B. Peristalsis through the descending colon is usually active C. This is the location of the pyloric sphincter D. Vascular sounds are the best heard in this area

A

Auscultation of the abdomen may reveal bruits of the _____ arteries. A. Aortic, renal, iliac, and femoral B. Jugular, aortic, carotid, and femoral C. Pulmonic, aortic, and portal D. Renal, iliac, internal jugular, and basilic

A

Bell palsy is characterized by: A. Unilateral paralysis of complete half of the face B. Bulging eyeballs C. A face that appears masklike D. A puffy edematous face

A

Cephalhematoma is associated with: A. Subperiosteal hemorrhage B. Increased intracranial pressure C. Down syndrome D. Cerebral palsy

A

Cessation of menses is known as A. menopause. B. menarche. C. adnexa. D. salpingitis.

A

Claudication is caused by A. arterial insufficiency. B. venous insufficiency. C. stasis ulcerations. D. varicose veins.

A

During a visit for a school physical, the 13-year-old girl being examined questions the asymmetry of her breasts. What is the nurse's best response? A. "One breast normally may grow faster than the other during development." B. "I will give you a referral for a mammogram." C. "You will probably have fibrocystic disease when you are older." D. This may be an indication of hormonal imbalance. We will check again in 6 months."

A

During an initial home visit, the patient's temperature is noted to be 97.4F. How would you interpret this? A. It cannot be evaluated without knowledge of the person's age B. It is below normal. The person should be assessed for possible hypothermia C. It should be retaken by the rectal route, because this best reflects core body temperature D. It should be reevaluated at the next visit before. decision is made

A

During an interview, a patient denies having any anxiety. The patient frequently changes position in the chair, hold his arms folded tight against his chest, and has little eye contact with the interviewer. The interviewer should: A. Use confrontation to bring the discrepancy between verbal and nonverbal behavior to the patient's attention B. Proceed with the interview. Patient are usually truthful with healthcare practitioners C. Make a mental note to discuss the behavior after the physical examination is completed D. Proceed with the interview and examination as outlined on the agency assessment form. The patient's behavior is appropriate for the circumstances

A

During auscultation of the lungs, the nurse expects decreased breath sounds to be heard in which situation? A. When the bronchial tree is obstructed B. When adventitious sounds are present C. In conjunction with whispered pectoriloquy D. In conditions of consolidation, such as pneumonia

A

During the examination of a full-term male newborn, a finding requiring investigation would be: A. An absent testes B. A meatus centered at the tip of the penis C. A wrinkled scrotum D. A penis 2 to 3 cm in length

A

During the examination of the lower extremities, you are unable to palpate the popliteal pulse. You should: A. Proceed with the examination. It is often impossible to palpate this pulse B. Refer the patient to a vascular surgeon for further evaluation C. Schedule the patient for a venogram D. Schedule the patient for an arteriogram

A

During the first trimester of pregnancy the uterus becomes globular in shape, softens, and flexes easily over the cervix. What term describes this change? A. Hegar sign B. Goodell sign C. Epulis of pregnancy D. Chadwick sign

A

During the interview, the nurse notes that the client is confused as to day and time. This would be an indicator of A. Client's orientation B. Client's affect and mood C. Client's willingness to cooperate D. Client's level of anxiety

A

Dysmenorrhea is A. pain associated with menstruation. B. pain with urination. C. pain with defecation. D. painful intercourse.

A

Examination of the shoulder includes 4 motions. These are: A. Forward flexion, internal rotation, abduction, and external rotation B. Abduction, adduction, pronation, and supination C. Circumduction, inversion, eversion, and rotation D. Elevation, retraction, protraction, and circumduction

A

Fetal heart tones are best auscultated over the fetus's: A. Back B. Abdomen C. Shoulder D. Head

A

Foods that may be beneficial to reduce the risk for colon cancer include: A. Foods high in fiber B. Foods low in fat C. Foods high in protein D. Foods high in carbohydrates

A

Functions of the skin include A. temperature regulation. B. secretion of a drying substance called sebum. C. production of vitamin C. D. production of new cells by melanocytes.

A

Gynecomastia is: A. Enlargement of the male breast B. Presence of mast cells in the male breast C. Cancer of the male breast D. Presence of supernumerary breast on the male chest

A

Heberden and Bouchard nodes are hard and nontender and are associated with: A. osteoarthritis B. metacarpophalangeal bursitis C. Dupuytren contracture D. rheumatoid arthritis

A

In which of the following groups does osteoporosis primarily occur? A. Postmenopausal white women B. Asian men C. African Americans D. American Indians

A

It is normal to palpate a few lymph nodes in the neck of a healthy person. What are the characteristics of these nodes? A. Mobile, soft, nontender B. Large, clumped, tender C. Matted, fixed, tender, hard D. Matted, fixed, nontender

A

Kyphosis of the spine is common with aging. To compensate, older adults: A. Extend their heads and jaws forward B. Shuffle C. Stiffen their gait D. Increase their center of gravity

A

Methods to enhance abdominal wall relaxation during examination include: A. positioning the patient with the knees bent B. examining painful areas first C. having the patient place arms above the head D. cool environment

A

Moles on the abdomen: A. are common. B. are uncommon. C. require a biopsy. D. are no cause for concern.

A

Neuropathic pain implies an abnormal A. processing of the pain message. B. degree of pain interpretation. C. modulation of pain signals. D. transmission of pain signals.

A

Normal cervical nodes are: A. Smaller than 1 cm B. Warm to palpation C. Fixed D. Firm

A

Of what does the patient believe the amulet is protective? A. The evil eye B. Being kidnapped C. Exposure to bacterial infection D. An unexpected fall

A

On examination of the mouth of an American Indian, the examiner notices the presence of a bifid uvula. How should this finding be interpreted? A. This is an expected variation associated with this individual. B. This may indicate the presence of oral cancer. C. This is rare and indicates other congenital anomalies may be present. D. This condition is frequently associated with cleft palate.

A

One of the leg's deep veins is the A. popliteal. B. great saphenous. C. tibial. D. small saphenous.

A

What is the first step in the assessment of any body system? A. Inspection B. Palpation C. Auscultation D. Percussion

A

Select the findings that most accurately describes appearance of a patient: A. Tense posture and restless activity. Clothing clean but not appropriate for season (e.g., patient wearing T-shirt and shorts in cold weather). B. Oriented x 3. Affect appropriate for circumstances. C. Alert and responds to verbal stimuli. Tearful when diagnosis discussed. D. Laughing inappropriately, orients x 3.

A

Sensorineural hearing loss may be related to: A. Gradual nerve degeneration B. Foreign bodies C. Impacted cerumen D. Perforated tympanic membrane

A

Shifting dullness is a test for: A. Ascites B. Splenic enlargement C. Inflammation of the kidney D. Hepatomegaly

A

Testing the deep tendon reflexes gives the examiner information regarding the intactness of the: A. reflex arc at specific levels in the spinal cord B. medulla C. corticospinal tract D. upper motor and lower motor neuron synaptic junction

A

The ability of the heart to contract independently of any signals or stimulation is due to A. automaticity. B. conduction. C. depolarization. D. repolarization.

A

The bell of the stethoscope is used: A. For soft, low-pitched sounds B. For high-pitched sounds C. To hold firmly against the skin D. To magnify sound

A

The best technique to put the client at ease during palpation portion of assessment would be for the nurse to A. Let the client know what you will be doing before palpating the area and touch the area before beginning the palpation B. Make sure the client is distracted and quickly do the palpation C. Explain each movement after completion D. Palpate known painful areas first

A

The bulge sign is a test for: A. Swelling in the suprapatellar pouch B. Carpal tunnel syndrome C. Heberden nodes D. Olecranon bursa inflammation

A

The dorsa of the hands are used to determine A. temperature. B. fine tactile discrimination. C. position of an organ. D. vibration.

A

The ejaculatory duct is A. the passage formed by the joining of the vas deferens and the seminal vesicle. B. a narrow tunnel inferior to the inguinal ligament. C. a narrow tunnel superior to the inguinal ligament. D. a muscular duct continuous with the epididymis.

A

The etiology of a pilonidal cyst is: A. a congenital disorder. B. a chronically inflamed gastrointestinal tract. C. trauma or irritant diarrheal stools. D. a tear in the superficial mucosa.

A

The examiner is assessing the extraocular muscles. Which of the following tests would be inappropriate? A. Confrontation test B. Corneal light reflex C. Cranial nerve III, IV, and VI testing D. Six cardinal positions of gaze

A

The examiner notices a fine tremor when the patient sticks out his or her tongue. What disorder is consistent with this finding? A. Hyperthyroidism B. Diabetic ketoacidosis C. Halitosis D. Alcoholism

A

The external structure of the ear is identified as the: A. Auricle. B. Auriga. C. Atrium. D. Aureole.

A

The first sign of puberty in boys is A. enlargement of the testes. B. pubic hair growth extending up the abdomen. C. an increase in penis size. D. the appearance of pubic hair.

A

The first sign of puberty in girls is A. breast and pubic hair development. B. rapid increase in height. C. the first menstrual cycle (menarche). D. axillary hair development.

A

The four layers of large, flat abdominal muscles form the: A. ventral abdominal wall B. rectus abdominis C. viscera D. linea alba

A

The frenulum is: A. Midline fold of tissue that connects the tongue to the floor of the mouth B. Anterior border of the oral cavity C. Arching roof of the mouth D. Free projection hanging down from the middle of the soft palate

A

The function of the nasal turbinates is to: A. Warm the inhaled air B. Detect odors C. Stimulate tear formation D. Lighten the weight of the skull bones

A

The functions of the musculoskeletal system include: A. protection and storage B. propulsion and preservation C. movement and elimination D. storage and control

A

The gastrocolic reflex is: A. peristaltic wave B. The passage of meconium in the newborn C. Another term for borborygmi D. Reverse peristalsis

A

The largest salivary gland is located: A. Within the cheek in front of the ear B. Beneath the mandible at the angle of the jaw C. Within the floor of the mouth under the tongue D. At the base of the tongue

A

The manubriosternal angle is: A. The articulation of the manubrium and the body of the sternum B. A hollow, U-shaped depression just above the sternum C. Also known as the breastbone D. A term synonymous with costochondral junction

A

The medical record indicates that a person has an injury to Broca's area. When meeting this person, you expect: A. Difficulty speaking B. Receptive aphasia C. Visual disturbances D. Emotional lability

A

The most appropriate introduction to use to start an interview with an older adult patient is A. "Mr. Jones, I want to ask you some questions about your health so that we can plan your care." B. "David, I am here to ask you questions about your illness; we want to determine what is wrong." C."Because so many people have already asked you questions, I will just get the information from the chart." D. "Mr. Jones, is it okay if I ask you several questions this morning about your health?"

A

The nurse caring for a client who is a long-term smoker is told in shift report that the client has clubbing of the fingers. To assess for clubbing, the nurse would instruct the client to do which of the following things? A. "Bring the backside of two corresponding fingers together." B. "Put your two index fingers tip to tip." C. "Hold your hands out straight with your palms sides down." D. "Place your two thumbs touching side by side."

A

The nurse documents the following findings for the behavioral portion of the general survey assessment, "patient demonstrates flat affect, lack of eye contact, hair not brushed, and strong body odor". The nurse should be concerned that the patient is which of the following? A. Depression B. Bulimia C. Dysarthria D. Seizures

A

The nurse hears dullness while percussing over the left lower quadrant. What would be the most appropriate question to ask the client? A. "When was your last bowel movement?" B. "Have you ever had splenomegaly?" C. "Do you have pain after eating?" D. "What is your normal diet?"

A

The nurse is assessing the patient's neck, which of the following is included for this area? A. Palpate cervical lymph nodes B. Grade tonsils if present C. Inspect the configuration D. Ask the patient to say "Ahh"

A

The nurse is auscultating a client's abdomen for bowel sounds and no sounds have been detected for at least two minutes. The nurse should A. Listen for at least three more minutes B. Document bowel sounds absent and identify the appropriate location C. Call the physician D. Document this finding as normal and move on to the next step of the physical assessment

A

The nurse is examining a client with a chief complaint of numbness and tingling in the hands. The nurse asks the client to bend the wrist downward and press the backs of the hands together. The client complains of numbness and tingling in the arms during the procedure. The nurse would suspect which of the following conditions in this situation? A. Carpal tunnel syndrome B. Crepitus of the wrists C. Dupuytren's contracture D. Arthritis of the wrists

A

The nurse is interviewing a male client with a chief complain of "feels like a bag of worms in my scrotum." The nurse would correctly suspect which of the following conditions? A. Varicocele B. Epididymitis C. Hernia D. Orchitis

A

The nurse is palpating a female patient's breasts during an examination. Which of these positions is most likely to make significant lumps more distinct during breast palpation? A. Supine with the arms raised over her head B. Sitting with the arms relaxed at her sides C. Supine with the arms relaxed at her sides D. Sitting with the arms flexed and fingertips touching her shoulders

A

The nurse is performing a vaginal examination on a pregnant client and notes the cervix is soft in texture and nontender. The nurse would correctly document which of the following conditions in this situation? A. Goodell's sign B. Chadwick's sign C. Piscecek's sign D. Hegar's sign

A

The nurse is performing sterognosis on a client. Which of the following instructions would the nurse provide for the client? A. "Identify the object in your hand with your eeys closed." B. "identify the number being traced in your hand with your eyes closed." C. "Tell me if you feel one or two objects touching you with your eyes closed." D. "Tell me what part of your body is being touched."

A

The nurse records that the patient's pulse is 3+ or full and bounding. Which of the following could be the cause? A. Anxiety B. Shock C. Bleeding D. Dehydration

A

The nurse will use which technique of assessment to determine the presence of crepitus, swelling, and pulsations? A. Palpation B. Inspection C. Percussion D. Auscultation

A

The position of the tympanic membrane in the neonate is more ________________, making it more difficult to visualize with the otoscope. A. horizontal B. perpendicular C. oblique D. vertical

A

The practitioner, entering the examining room to meet a patient for the first time, states: Hello, I'm M.M,. and I'm here to gather some information from you and to perform your examination. This will take about 30 minutes. D.D. is a student working with me. If it's all right with you, she will remain during the examination." Which of the following must be added to cover all aspects of the interview contract. A. A statement regarding confidentiality, patient costs, and the expectations of each person. B. the purpose of the interview and the role of the interviewer. C. Time and place of the interview and a confidentiality statement. D. An explicit purpose of the interview and a description of the physical examination, including diagnostic studies.

A

The structure that secretes a thin, milky alkaline fluid to enhance the viability of sperm is the: A. Prostate gland. B. Cowper gland. C. Bulbourethral gland. D. Median sulcus.

A

The tympanic membrane of a child with acute otitis media would be: A. Bulging with a distinct red color B. Flat and slightly pulled in at the center C. Mobile and would flutter with the Valsalva maneuver D. Shiny and translucent, with a pearly gray color

A

The use of euphemisms to avoid reality or to hide feelings is known as A. avoidance language. B. distancing language. C. sympathetic language. D. ethnocentric language.

A

Three of these findings can be assessed with an obstetric ultrasound. Which one cannot? A. Thickness of the uterine wall B. Fetal position C. Placental location D. Amniotic fluid volume

A

To assess for early jaundice, you will assess: A. Sclera and hard palate B. Nail beds C. Lips D. All visible skin surfaces

A

To examine a toddler, the nurse should A. allow the child to sit on the parent's lap. B. ask the child to decide whether parents or siblings should be present. C. remove the child's clothing at the beginning of the examination. D. perform the assessment from head to toes.

A

To perform an accurate assessment of respirations, the examiner should A. count for 30 seconds after completing a pulse assessment and multiply by two. B. inform the person of the procedure and count for 1 minute. C. assess respirations for a full 2 minutes if an abnormality is suspected. D. count for 15 seconds while keeping fingers on the pulse and then multiply by four.

A

Toilet training (for bowel movements) in children: A. may begin when the nerves in the rectal area are fully myelinated B. should start after 4 years of age C. can be successful as early as 12 months of age D. should begin after the gastrocolic reflex disappears.

A

Using Nagele rule, the estimated date of delivery (EDD) if a women's last menstrual period started on January 13 is: A. 1/13 + 7 = 20, 20-3 months, EDD = October 29 B. 1/13 + 10 = 23, 23-3 months, EDD = October 18 C. 1/13 + 14 = 27, 27-3 months, EDD = September 30 D/ 113 - 7 = 6, 6-3 months, EDD = October 6

A

Viewing the world from another person's inner frame of reference is called A. empathy. B. clarification. C. reflection. D. sympathy.

A

Visual acuity is assessed with: A. The Snellen eye chart B. An ophthalmoscope C. The Hirschberg test D. The Confrontation test

A

What does active listening NOT include? A. Taking detailed notes during the interview B. Watching for clues in body language C. Repeating statements back to the person to make sure you have understood D. Asking open-ended questions to explore the person's perspective

A

What is the best description of "review of systems" as part of the health history? A. The evaluation of the past and present health state of each body system B. A documentation of the problem as described by the patient C. The recording of the objection findings of the practitioner D. A statement that describes the overall health state of the patient

A

When using the ophthalmoscope, the interruption of the red reflex occurs when: A. There is opacity in the cornea or lens B. The patient has a pathologic process of the optic tract C. The blood vessels are tortuous D. The pupils are constricted

A

When using the ophthalmoscope, you would: A. Remove your own glasses and approach the patient's left eye with your left eye B. Leave the light on in the examining room and remove glasses from the patient C. Remove glasses and set the diopter setting at 0 D. Use the smaller white light and instruct the patient to focus on the ophthalmoscope

A

Which best describes evidence-based nursing practice? A. Combining clinical expertise with the use of nursing research to provide the best care for patients while considering the patient's values and circumstances B. Appraising and looking at the implications of one or two articles as they relate to the culture and ethnicity of the patient C. Completing a literature search to find relevant articles that use nursing research to encourage nurses to use good practices D. Finding value-based resources to justify nursing actions when working with patients of diverse cultural backgrounds

A

Which characteristic of the prostate gland would the nurse recognize as an abnormal finding while palpating the prostate gland through the rectum? A. Tenderness to palpation B. Elastic and rubbery consistency C. Heart shaped D. Palpable central groove

A

Which culture would describe illness as hot and cold imbalance? A. Asian-American heritage B. African-American heritage C. Hispanic-American heritage D. American Indian heritage

A

Which description would differentiate a split S2 from S3? A. S3 is lower pitched and is heard at the apex B. S2 is heard at the left lower sternal border C. The timing S2 varies with respirations D. S3 is heard at the base; the timing varies with respirations

A

Which factor is identified as a priority influence on a patient's health status? A. Poverty B. Lifestyle factors C. Legislative ction D. Occupation status

A

Which finding would require immediate action by the nurse if found during the physical assessment? A. Oxygen saturation of 88% B. Heart rate of 60 beats per minute C. Respirations of 20 D. Systolic blood pressure of 152 mm Hg

A

Which is the most common bacterial sexually transmitted infection in the United States? A. Chlamydia B. Gonorrhea C. Tichomoniasis D. Syphilis E. Bacterial vaginosis

A

Which is true regarding cluster headaches? A. They may be precipitated by alcohol and daytime napping. B. Their usual occurrence is two per month, each lasting 1 to 3 days. C. They are characterized as throbbing. D. They tend to be supraorbital, retro-orbital, or frontotemporal.

A

Which of the following assessment best confirms symmetric chest expansion? A. Placing hands on the posterolateral chest wall with thumbs at the level of T9 or T10 and then sliding the hands up to pinch a small fold of skin between the thumbs B. Inspection of the shape and configuration of the chest wall C. Placing the palmar surface of the fingers of one hand against the chest and having the person repeat the words "ninety-nine" D. Percussion of the posterior chest

A

Which of the following guidelines may be used to identify which heart sound is S1? A. S1 coincides with the carotid artery pulse. B. S1 coincides with the A wave of the jugular venous pulse wave. C. S1 coincides with the Q wave of the QRS electrocardiogram complex. D. S1 is louder than S2 at the base of the heart.

A

Which of the following is (are) normal, common finding (s) on inspection and palpation of the vulva and perineum? A. Labia majora that are wide apart and gaping B. Palpable Bartholin glands C. Clear, thin discharge from paraurethral glands D. Bulging at introitus during Valsava maneuver

A

Which of the following is an appropriate position to have the patient assume when auscultating for extra heart sounds or murmurs? A. Roll toward the left side B. Recumbent position C. Trendelenburg position D. Roll toward the right side

A

Which of the following phrases contain objective data? A. "Hispanic female sitting on examination table holding a towel on her forehead." B. "I am six-years-old and I'm here because I bumped my head." C. "My head is bleeding and it won't stop." D. "Client states that she fell off her bicycle."

A

Which of the following questions would be important to include when completing a subjective assessment of the breasts for a female patient to determine whether she would be at increased risk for breast cancer? A. "Have you taken oral contraceptives?" B. "Have you ever had a rash on or around your breasts? C. "Have you ever been taught breast self-examination?" D. "Have you noticed any enlargement or fullness in the breasts?"

A

Which of the following statements describing a headache would warrant an immediate referral? A. "I have never had a headache like this before; it is so bad I can't function." B. "This is the worst headache I've had since puberty." C "This is the worst migraine of my life." D. "I have had daily headaches for years."

A

Which of the following statements made by the interviewer would be an appropriate response? A. "Tell me what you mean by 'bad blood.'" B. "If I were you, I would have the surgery." C. "I know just how you feel." D. "Why did you wait so long to make an appointment?"

A

Which of the following tests provides a precise quantitative measure of hearing? A. Audiometer test B. Tuning fork tests C. Whispered voice test D. Romberg test

A

Which priority action should the nurse take when performing an initial assessment of pain status of a patient who is receiving pain control via patient-controlled analgesia (PCA)? A. Ask the patient to rate his or her pain on a numeric scale of 1 to 10. B. Determine when the patient last used the PCA button by looking at patient history on the PCA. C. Confirm that the correct intravenous (IV) fluid is hanging. D. Position the patient for comfort.

A

Which retinal structures can be viewed through the ophthalmoscope? A. The optic disc, the retinal vessels, the general background, and the macula B. The cornea, the lens, the choroid, snd the ciliary body C. The optic papilla, the sclera, the retina, and the iris D. The pupil, the sclera, the ciliary body, and the macula

A

Which statement best describes religion? A. An organized system of beliefs concerning the cause, nature, and purpose of the universe B. Belief in a divine or superhuman spirit to be obeyed and worshiped C. Affiliation with one of the 2300 recognized religions in the United States D. The following established rituals, especially in conjunction with health-seeking behaviors

A

While percussing over the liver, the nurse finds that the liver span is approximately 7 cm. This would be documented as: A. a normal finding. B. enlargement of the liver. C. displacement due to ascites. D. displacement as a result of respiratory disease.

A

While performing an assessment on a healthy 4-year-old, the nurse palpates bilateral superficial cervical lymph nodes, which are soft, mobile, nontender, and less than one cm in diameter. The nurse's response to this finding would be based on which of the following? A. Palpable lymph nodes in a child can be a normal finding. B. Palpable lymph nodes are always considered an abnormal finding. C. Lymph nodes in children tend to be smaller than those in adults and would only be palpable with infection or other causes of inflammation. D. In the absence of other symptoms, palpable lymph nodes would most likely be a reflection of lymphatic disease.

A

Why is it important to match the appropriate size of blood pressure cuff to the person's arm and shape and not to the person's age? A. Using a cuff that is too narrow will give a false reading that is high B. Using a cuff that is too wide will give a false reading that is low C. Using a cuff that is too narrow will give a false reading that is too low D. Using a cuff that is too wide will give a false reading that is high

A

You are assessing an African-American patient and note a flat, 3-cm, nontender, grayish-white lesion on the left buccal mucosa. Which of the following is most likely? A. The lesion is a leukoedema and is common in darkly pigmented persons B. This is the result of hyperpigmentation and is normal C. This is torus palatinus and would normally only be found in smokers D. This type of lesion is indicative of cancer and should be tested immediately

A

You are assessing capillary refill. The room is warm. Which finding would be considered normal? A. Less then or equal to 1 second B. Greater than 2 seconds C. 2 to 3 seconds D. Time is not significant as long as color returns

A

You are leading a discussion of the planned activities for the day at an adult living center and state, "We will be having snacks at 9:30 and lunch will be at noon." One of the participants responds in a very monotone manner, "Snacks at 9:30, lunch at noon." This patient is exhibiting signs of: A. Echolalia B. Confabulation C. Flight of ideas D. Neologisms

A

You are providing health promotion teaching for a 40-year-old woman. What is the current recommendation for women 40 to 44 years of age and older for breast cancer screening with mammography? A. May begin every year B. Every 2 years C. Twice a year D. Only baseline examination needed unless the woman has symptoms

A

You are reviewing concepts related to steps in the nursing process for determining prioritization and developing patient outcomes. To what are these actions attributed? A. Planning B. Assessment C. Implementation D. Diagnosis

A

You auscultate a patient to rule out a pericardial friction rub. Which assessment technique is most appropriate? A. Listen with the diaphragm, patient sitting up leaning forward, breath held in expiration B. Listen using the bell with the patient leaning forward C. Listen at the base during normal respiration D. Listen with the diaphragm, patient turned to the left side

A

You have reason to question the reliability of the information being provided by a patient. One way to verify the reliability within the context of the interview is to: A. Rephrase the same questions later in the interview B. Review the patient's previous medical record C. Call the person identified as the emergency contact to verify the data provided D. Provide the patient with a printed history to complete and then compare the data provided

A

____________________ is exhibiting an accurate understanding of the other person's feelings within a communication context. A. Empathy B. Liking others C. Facilitation D. A nonverbal listening technique

A

the congenital displacement of the urethral meatus to the inferior surface of the penis is: A. Hypospadias B. Epispadias C. Hypoesthesia D. Hypophysis

A

When gathering information relative to a complete health assessment, the nurse should include which of the following in the decision-making process? (Select all that apply.) A. Use line drawings to explain and record pertinent findings. B. Gather needed equipment before the start of the health assessment. C. Write down "word for word" what the patient says as evidence of reliable documentation. D. Treat the health assessment as a legal document.

A, B, & D

When preparing to complete thorax and lung objective assessment of an infant or child, the nurse should do which of the following? (Select all that apply.) A. Allow the caregiver to hold the infant. B. If infant is sleeping, perform auscultation before doing any other objective assessments. C. There is no need to change the language you use when telling the young child to breathe in and out. D. Warm the stethoscope between your hands before placing on the infant. E. Document a barrel chest shape as expected findings for a 7-year-old child.

A, B, & D

Which of the following will help the student nurse improve his or her health assessment techniques? (Select all that apply.) A. Repeatedly rehearse all appropriate assessment techniques. B. Watch videos of expert health care professionals performing health assessment techniques. C. Stand in the corner of the patient's room while the nurse completes a health assessment. D. Follow a routine sequence. E. Work on paperwork during clinical.

A, B, & D

When considering cultural competence, the nurse must develop knowledge of discrete areas to understand the health care needs of others. These discrete areas include understanding of (Select all that apply.) A. his or her own heritage. B. cultural and ethnic values. C. the heritage of the health care system. D. the heritage of the nursing profession. E. the heritage of the patient.

A, C, D, & E

Which of the following factors control a person's blood pressure? (Select all that apply.) A. Cardiac output B. Age C. Emotions D. Viscosity E. Vascular resistance

A, D, & E

A 70-year-old woman reports dry mouth. The most frequent cause of this is: A. The aging process B. Related to medications she may be taking C. The use of dentures D. Related to a diminished sense of smell

B

During auscultation of breath sounds, the examiner should A. listen with the bell of the stethoscope. B. compare sounds on the left and right sides. C. instruct the patient to breathe in and out through the nose. D. listen only to the posterior chest for adventitious sounds.

B

Select the best description of the uterus: A. Anteverted, round, asymmetric organ B. Pear-shaped, thick-walled organ flattened anteroposteriorly C. Retroverted, almond-shaped, asymmetric organ D. Midposition, thick-walled, oval organ

B

Moving a limb away from the midline of the body

Abduction

Moving a limb toward the midline of the body

Adduction

A 2-year-old child comes to the clinic for a health examination. A common finding for this age group is: A. Kyphosis B. Lordosis C. Scoliosis D No deviation is normal

B

A 20-year-old man has indicated that he does not perform a testicular self-examination. One of the facts that should be shared with him is that testicular cancer, although rare, does occur in men: A. Younger than 15 years B. 15 to 34 years of age C. 35 to 55 years of age D. 55 years and older

B

A 65-year-old patient with a history of heart failure comes to the clinic with complaints of "being awakened from sleep with shortness of breath." Which action by the nurse is most appropriate? A. Obtain a detailed history of the patient's allergies and history of asthma. B. Assess for other signs and symptoms of paroxysmal nocturnal dyspnea. C. Tell the patient to sleep on his or her right side to facilitate ease of respirations. D. Assure the patient that this is normal and will probably resolve within the next week.

B

A flat macular hemorrhage is called a(n) A. petechiae. B. purpura. C. ecchymosis. D. hemangioma.

B

A patient admitted to the hospital with asthma has the following problems identified based on an admission health history and physical assessment. Which problem is a first-level priority? A. Ineffective self-health management B. Impaired gas exchange C. Readiness for enhanced spiritual well-being D. Risk for infection

B

A patient has hypoactive bowel sounds. The nurse knows that a potential cause of hypoactive bowel sounds is: A. diarrhea. B. peritonitis. C. laxative use. D. gastroenteritis.

B

A patient has soft, moist, fleshy, painless papules around the anus. The examiner suspects this condition is: A. HSV-2 B. HPV C. Gonorrhea D. Peyronie disease

B

A patient is in the emergency department with nausea and vomiting. Which would you include in the database? A. A complete health history and full physical examination B. A diet and GI history C. Previously identified problems D. Start collection of data in conjunction with lifesaving measures

B

A patient is known to be blind in the left eye. What happens to the pupils when the right eye is illuminated by a penlight beam? A. No response in both pupils B. Both pupils constrict C. Right pupil constricts, left has no response D. Left pupil constricts, right has no response

B

A patient presents with acute pain of the abdomen. After the initial examination, how would you proceed? A. Withhold analgesic until diagnostic testing is completed B. Give pain medication as ordered C. Withhold analgesic until pain subsides D. Determine what type of pain it is and proceed accordingly

B

A patient with a head injury has clear, watery drainage from the ear. How should you proceed? A. Place a cotton ball loosely at the entrance to the ear canal B. Consider possibility of basal skull fracture and refer immediately C. Perform pneumatic otoscopy to assess for drum hypomobility D. Assess for the presence of a tympanostomy tube in the ear

B

A patient's pulse with an amplitude of 3+ indicates: A. Irregular, with 3 premature beats B. Increased, full C. Normal D. Wark

B

A positive Murphy sign is best described as: A. The pain felt when the examiner's hand is rapidly removed from an inflamed appendix B. Pain felt when taking a deep breath when the examiner's fingers are on the approximate location of the inflamed gallbladder C. A sharp pain felt by the patient when one hand of the examiner is used to thump the other at the costovertebral angle D. This is not a valid examination technique

B

A pregnancy test measures levels of A. estrogen and progesterone. B. human chorionic gonadotropin. C. human placental lactogen. D. alpha-fetoprotein.

B

A pregnant adolescent is medically at risk for: A. Poor weight gain, preeclampsia, thyroiditis, miscarriage B. Poor weight gain, preeclampsia, prolonged labor C. Stress, abuse, inadequate housing, inadequate education D. Miscarriage, hypothyroidism, poor weight gain

B

A student nurse has been assigned to teach fourth graders about hygiene. While preparing, the student nurse adds information about the sweat glands. Which of the following should be included while discussing this topic? A. There are two types of sweat glands: eccrine glands and sebaceous glands. B. Bacteria that normally lives on the skin interacts with apocrine sweat to produce body odor. C. The evaporation of sweat, a dilute saline solution, increases body temperature. D. Eccrine glands produce sweat and are mainly located in the axillae, anogenital area, and navel.

B

A woman has just been diagnosed with HPV or genital warts. The nurse should counsel her to receive regular examinations because this virus makes her at a higher risk for _______ cancer. A. Uterine B. Cervical C. Ovarian D. Endometrial

B

After assessing the patient's pulse, the practitioner determines it to be "normal." This would be recorded as: A. 3+ B. 2+ C. 1+ D 0

B

An 18-month-old child is brought into the clinic for a health screening visit. To assess the height of the child: A. Use a tape measure B. Use a horizontal measuring board C. Have the child stand on the upright scale D. Measure arm span to estimate height

B

An adhesion of the prepuce to the head of the penis, making it impossible to retract, is: A. paraphimosis B. phimosis C. smegma D. dyschezia

B

An adolescent male is brought to the emergency department with complaints of excruciating pain in his left testicle. Which of the following would be the nurse's most appropriate action? A. Perform a focused assessment. B. Notify the emergency department physician immediately. C. Document pain assessment and notify physician when he or she is available. D. Tell the adolescent, everything will be fine and there's nothing to worry about.

B

An anorectal fistula is usually caused by: A. a tear in the superficial mucosa B. a chronically inflamed gastrointestinal tract C. daily use of laxatives D. trauma from passing hard stools

B

An expected postmenopausal breast change is A. decreased fibrous connective tissue. B. decreased breast size. C. increased fatty tissue. D. increased glandular tissue.

B

An increase in the transverse diameter of the chest cage in a pregnant woman is due to a(n) A. increase in surfactant. B. increase in estrogen. C. increase in tidal volume. D. compensatory increase in respiratory parenchyma.

B

Aphasia is best described as A. a disturbance in executive functioning (planning, organizing, sequencing, abstracting). B. a language disturbance in speaking, writing, or understanding. C. the impaired ability to recognize or identify objects despite intact sensory function. D. the impaired ability to carry out motor activities despite intact motor function.

B

Arteriosclerosis is caused by: A. Deposition of fatty plaques on the intima of the arteries B. Loss of elasticity of the walls of blood vessels C. Loss of lymphatic tissue that occurs in the aging process D. Progressive enlargement of the intramuscular calf veins

B

Assessing a patient's skin turgor is done to assess which clinical finding? A. Edema B. Dehydration C. Vitiligo D. Scleroderm

B

Atrial systole occurs: A. During ventricular systole B. During ventricular diastole C. Concurrently with ventricular systole D. Independently of ventricular function

B

Binaural interaction at the level of the brainstem permits: A. Amplification of sound. B. Identification and location of the direction of the sound. C. Direction of sound toward the appropriate conduction pathway. D. Interpretation of sound.

B

Bleeding into the periosteum during birth is known as: A. Caput succedaneum B. Cephalhematoma C. Molding D. Craniosynostosis

B

Bloody show occurs with A. implantation. B. the onset of labor. C. ovulation. D. fetal movement.

B

Cerebellar function is tested by: A. muscle strength assessment. B. performance of rapid alternating movements. C. the Phalen maneuver. D. superficial pain and touch assessment.

B

Children usually come for health care with a caregiver. At about what age should the interviewer begin to question the child himself or herself regarding presenting symptoms? A. 5 years B. 7 years C. 9 years D. 11 years

B

Conjunctivitis is associated with: A. Absent red reflex B. Reddened conjunctiva C. Impairment of vision D. Fever

B

Data collection for the general survey begins A. at the beginning of the physical examination. B. at the first encounter. C. while taking vital signs. D. during the mental status examination.

B

During a breast examination on a female patient, the nurse notices that the nipple is flat, broad, and fixed. The patient states it "started doing that a few months ago." This finding suggests: A. Dimpling. B. Retracted nipple. C. Nipple inversion. D. Deviation in nipple pointing.

B

During a breast examination you detect a mass. Which of the following is most consistent with cancer rather than benign breast disease? A. Round, firm, well demarcated B. Irregular, poorly, defined, fixed C. Rubbery, mobile, tender D. Lobular, clear margins, negative skin retraction

B

During a breast health interview, a patient states that she has noticed pain in her left breast. The nurse's most appropriate response to this would be: A. "Don't worry about the pain; breast cancer is not painful." B. "I would like some more information about the pain in your left breast." C. "Oh, I had pain like that after my son was born; it turned out to be a blocked milk duct." D. "Breast pain is almost always the result of benign breast disease."

B

During a neurologic examination, the tendon reflex fails to appear. Before striking the tendon again, you use the technique of: A. Two-point discrimination B. Reinforcement C. Vibration D. Graphesthesia

B

During an initial interview, the examiner says, "Mr. J., tell me what you do when your headaches occur?" This is an example of which type of information? A. The patient's perception of the problem B. Aggravating or relieving factors C. The frequency of the problem D. The severity of the problem

B

During an inspection of a patient's nares, a deviated septum is noted. What should you do next? A. Request a consultation with an ear, nose, and throat specialist B. Document the deviation in the medical record in case the person needs to be suctioned C. Teach the person what to do if a nosebleed should occur D. Explore further because polyps frequently accompany a deviated septum

B

During an interview with a patient diagnosed with a seizure disorder, the patient states, "I plan to be an airline pilot." If the patient continues to have the as a career goal after teaching regarding seizure disorders has been provided, you might question the patient's: A. Thought processes B. Judgment C. Attention span D. Recent memory

B

During the examination of a 30-year-old woman, she asks about "the 2 large moles" that are below her left breast. After examining the area, how do you respond? A. "I think you should be examined by a dermatologist." B. "This is a normal finding of supernumerary nipples that are not developed." C. "These are Montgomery gland, which are common." D. "Is there a possibility you are pregnant?"

B

During the otoscopic examination of a child younger than 3 years, the examine: A. Pulls the pinna up and back B. Pulls the pinna down C. Hold the pinna gently but firmly in its normal position D. Tilts the head slightly toward the examiner

B

Each culture has its own healers who usually A. own and operate specialty community clinics. B. cost less than traditional or biomedical providers. C. recommend folk practices that are dangerous. D. speak at least two languages.

B

Fine tactile discrimination is best achieved with the A. back of the hands and fingers. B. fingertips. C. base of the fingers. D. opposition of the fingers and thumb.

B

For a health assessment, which assessment technique will you use first? A. Palpation B. Inspection C. Percussion D. Auscultation

B

In order to obtain accurate subjective assessment data from a female adolescent, what would be the most appropriate action by the nurse? A. Stand when asking questions. B. Ask parent/caregiver to step out of the room and return to the lobby. C. Share what life was like for the nurse at the patient's age. D. Ask questions such as, "you're not sexually active, right?"

B

In placing a finger on either side of the cervix and moving it side to side, you are assessing: A. The diameter of the fallopian tube B. Cervical motion tenderness C. The ovaries D. The uterus

B

Inspiration is primarily facilitated by which of the following muscles? A. Internal intercostal and abdominis B. Diaphragm and intercostal C. Trapezius and sternomastoids D. Diaphragm and rectus abdominis

B

Knowledge of the use of personal space is helpful for the health care provider. Personal distance is generally considered to be: A. 0 to 1.5 feet B. 1.5 to 4 feet C. 4 to 12 feet D. 12 or more feet

B

Lymphedema is A. an inflammation of the vein associated with thrombus formation. B. the swelling of an extremity caused by an obstructed lymph channel. C. a thickening and loss of elasticity of the arterial walls. D. the indentation left after the examiner depresses the skin over swollen edematous tissue.

B

Nonverbal communication is the primary form of communication for which group of individuals? A. Older adults B. Infants C. Preschoolers D. Adolescents

B

Normal stool is described as: A. Black in color and tarry in consistency B. Brown in color and soft in consistency C. Clay-colored and dry in consistency D. Varies depending on the individual's diet

B

One of the causes of visual impairment in aging adults includes: A. Strabismus B. Glaucoma C. Amblyopia D. Retinoblastoma

B

One of the purposes of the paranasal sinuses is to A. amplify sound B. lighten the weight of the skull bones C. warm and moisten the inspired air D. augment the sensory sensation of smell

B

One way to assess cognitive function and to screen dementia is with: A. The Proverb Interpretation Test B. The Mini-Cog C. The Denver II D. The Older Adult Behavioral Checklist

B

Pain issues should be anticipated in a cognitively impaired older adult with a history of: A. Constipation B. Peripheral vascular disease C. COPD D. Parkinson's disease

B

Parents or caregivers accompany children to the health care setting. Starting at ___ years of age, the interviewer asks the child directly about his or her presenting symptoms. A. 11 B. 7 C. 9 D. 5

B

Pronation and supination of the hand and forearm are the result of the articulation of the: A. Scapula and clavicle B. Radius and ulna C. Patella and condyle of fibula D. Femur and acetabulum

B

Pyloric stenosis is a(n): A. abnormal opening in the pyloric sphincter. B. congenital narrowing of the pyloric sphincter. C. inflammation of the pyloric sphincter. D. abnormal enlargement of the pyloric sphincter.

B

Raynaud phenomenon occurs: A. When the patient's extremities are exposed to heat and compression B. In the hands and feet as a result of exposure to cold, vibration, and stress C. After removal of lymph nodes or damage to lymph nodes and channels D. As a result of leg cramps due to excessive walking or climbing stairs

B

Select the best description of an accurate assessment of a patient's pulse A. Count for 15 seconds if the pulse is regular B. Begin counting with zero; count for 30 seconds C. Count for 30 seconds and multiply by 2 for all cases D. Count for 1 full minute begin counting with zero

B

Select the best description of bronchovasicular breath sounds. A. High-pitched, of longer duration on inspiration than expiration B. Moderate-pitched, inspiration equal to expiration C. Low-pitched, inspiration greater than expiration D. Rustling sound, like the wind in the trees

B

Select the best description of the secretion of the eccrine glands. A. Thick, milky B. Dilute saline solution C. Protective lipid substance D. Keratin

B

Select the best description of the tricuspid valve. A. left semilunar valve B. right atrioventricular valve. C. left atrioventricular valve. D. right semilunar valve.

B

The Glasgow Coma Scale (GCS) is divided into three areas. They include: A. Pupillary response, a reflex test, and assessing pain B. Eye opening, motor response to stimuli, and verbal response C. Response to find touch, stereognosis, and sense of position D. Orientation, rapid alternating movements, and the Romberg test

B

The abdomen normally moves with breathing until the age of ____ years. A. 14 B. 7 C. 75 D. 4

B

The best description of the pitch of a sound wave obtained by percussion is: A. The intensity of the sound B. The number of vibrations per second C. The length of time and note lingers D. The overtones of the note

B

The divisions of the spinal vertebrae include: A. cervical, thoracic, scaphoid, sacral, and clavicular. B. cervical, thoracic, lumbar, sacral, and coccygeal. C. cervical, lumbar, iliac, synovial, and capsular. D. scapular, clavicular, lumbar, scaphoid, and fasciculi.

B

The examiner is palpating the apical impulse. Which is a normal-sized impulse? A. Less than 1 cm B. Approximately 1 x 2 cm C. 3 cm D. Varies depending on the size of the person

B

The examiner records "positive consensual light reflex." This indicates: A. Convergence of the axes of the eyeballs B. Simultaneous constriction of the other pupil when one eye is exposed to bright light C. Reflex direction of the eye toward an object attracting a person's attention D. Adaptation of the eye for near vision

B

The examiner should use handwashing instead of an alcohol-based hand rub A. if the patient is HIV positive. B. if the patient has an infection with Clostridium difficile. C. if the patient has an infection with Mycobacterium tuberculosis. D. if the patient has an infection with hepatitis B virus.

B

The expert nurse differs from the novice nurse by acting without consciously thinking about the actions. This is referred to as: A. Deductive reasoning B. Intuition C. The nursing process D. Focused assessment

B

The extraocular muscles consist of four straight or ________ muscles and two slanting or ______ muscles. A. rectilinear; diagonal B. rectus; oblique C. palpebral; conjugate D. superior; inferior

B

The extrapyramidal system is located in the: A. medulla B. basal ganglia C. hypothalamus D. cerebellum

B

The group of axillary lymph nodes that drains the other three groups of nodes is the A. anterior nodes. B. central nodes. C. posterior nodes. D. lateral nodes.

B

The isthmus of the thyroid gland lies just below the: A. Mandible B. Cricoid cartilage C. Hyoid cartilage D. Thyroid cartilage

B

The layer of skin containing the nerves and hair follicles is the A. Adipose B. Dermis C. Epidermis D. Apocrine

B

The mental status examination A. should be completed at the end of the physical examination. B. assesses mental health strengths and coping skills and screens for any dysfunction. C. is usually not assessed in children younger than 2 years of age. D. will not be affected if the patient has a language impairment.

B

The normal color of the optic disc is: A. Red B. Creamy yellow-orange to pink C. Creamy pink D. Creamy red to yellow-orange

B

The nurse auscultates the pulmonic valve area in which region? A. Second right interspace B. Second left interspace C. Left lower sternal border D. Fifth interspace, left midclavicular line

B

The nurse has completed a peripheral vascular assessment. Which of the following findings would he or she document as expected findings? A. Capillary refill <5 seconds. B. Radial pulses 2+ with regular rate and rhythm bilaterally C. Right ankle 1+ edema with no perceptible swelling of the leg. D. Feet pale and cool to touch.

B

The sensation of vertigo may indicate: A. Otitis media B. Pathology in the semicircular canals C. Pathology in the cochlea D. 4th cranial nerve damage

B

The nurse is conducting a health fair for older adults. Which statement is true regarding vital sign measurements in aging adults? A. The pulse is more difficult to palpate because of the stiffness of the blood vessels B. An increased respiratory rate and shallower inspiratory phase are possible findings C. A decreased pule pressure occurs from changes in systolic and diastolic blood pressures D. Changes in the body's temperature regulatory mechanisms leave the older adult more likely to develop a fever

B

The nurse is examining a third-trimester pregnant client who is complaining of lower-back pain. The nurse notes a slight lordosis and waddling gait in the client. The nurse would correctly choose which of the following actions in this situation? A. Notify the physician of the findings B. Document the findings as normal C. Ask the client if she has been doing heavy lifting D. Tell the client to go on bedrest

B

The nurse is interviewing a client in the first trimester of pregnancy who reports she has not felt the baby move yet. The nurse would correctly respond with which of the following choices? A. "You should not be concerned." B. "Fetal movement does not occur until the 18th week." C. "Have you lost a baby before?" D. "We will listen for the heartbeat today."

B

The nurse is interviewing a female client who reports absence of menstruation and breast tenderness. The nurse would document these as which classification of signs of pregnancy? A. Probable B. Presumptive C. Positive D. Objective

B

The nurse is palpating a pregnant client's thyroid and finds that she has slight enlargement of the gland. The nurse appropriately documents A. this condition as Grave's disease B. the size, shape, and location of the finding C. this finding as normal D. pathological hyperthyroidism

B

The nurse is palpating the scrotum of a male client and detects a hardened area in the right side of the scrotal sac. The nurse would correctly choose which of the following actions? A. Ask the client about voiding patterns B. Use a light to perform transillumination C. Notify the physician of the finding D. Ask the client about sexual practices

B

The nurse is palpating the sinus areas. If the findings are normal, then the patient should report which sensation? A. No sensation B. Firm pressure C. Pain during palpation D. Pain sensation behind eyes

B

The nurse is percussing over the client's bladder and notes a dull tone. The nurse understands this to represent A. An empty bladder B. A full bladder C. Percussion over one of the kidneys D. Air trapped in the intestines

B

The nurse is performing a neurological assessment on a client experiencing anosmia. The nurse would suspect cranial nerve involvement in which of the following? A. Optic, cranial nerve II B. Olfactory, cranial nerve I C. Oculomotor, cranial nerve III D. Trochlear, cranial nerve IV

B

The nurse is performing a vaginal exam on a female client and palpates a non-tender protrusion into the anterior vaginal wall. The nurse would suspect which of the following conditions in this situation? A. Prolapsed uterus B. Cystocele C. Rectocele D. Skene's gland infection

B

The nurse is using an otoscope to assess the nasal cavity. Which of these techniques is correct? A. Inserting the speculum at least 3 cm into the vestibule B. Avoiding touching the nasal septum with the speculum C. Gently displacing the nose to the side that is being examined D. Keeping the speculum tip medial to avoid touching the floor of the nares

B

The nurse questions the reliability of the history provided by the patient. One method to verify information within the context of the interview is to A. ask the patient if there is someone who could verify information. B. rephrase the same questions later in the interview. C. call a family member to confirm information. D. review previous medical records.

B

The nursing assistant takes the vital signs for the 12 patients on the unit. Who is responsible for interpreting the results? A. The unit manager must ensure that the nursing assistant reports any abnormal results. B. The registered nurse assigned to the patient(s) should interpret the vital signs. C. The nursing assistant should review the results for abnormalities. D. The charge nurse is responsible for reviewing the vital signs on all 12 patients.

B

The organ in the right upper quadrant of the abdomen is the: A. spleen B. liver C. sigmoid colon D. cecum

B

The palpebral fissure is the: A. Border between the cornea and sclera B. Open space between the eyelids C. Angle where the eyelids meet D. Visible on the upper and lower lids at the inner canthus

B

The tympanic membrane thermometer (TMT) A. senses the infrared emissions of the cerebral cortex. B. provides an accurate measurement of core body temperature. C. accurately measures temperature in 20 to 30 seconds. D. is not used in unconscious patients.

B

To elicit the Babinski reflex: A. Gently tap the Achilles tendon B. Stroke the lateral aspect of the sole of the foot from heel to across the ball C. Present a noxious odor to the person D. Observe the person walking heel to toe

B

To insert the speculum as comfortably as possible, the examiner: A. Opens the speculum slightly and inserts it in an upward direction B. Presses the introitus down with one hand and inserts the blades obliquely with the other C. Spreads the labia with one hand and inserts the closed speculum horizontally with the other D. Pushes down on the introitus and inserts the speculum in an upward direction

B

To use the technique of egophony; ask the patient to: A. Take several deep breaths and then hold for 5 seconds B. Say "eeeee" each time the stethoscope is moved C. Repeat the phrase "ninety-nine" each time the stethoscope is moved D. Whisper a phrase as auscultation is performed

B

What does the notation in a health record indicating the patient is a "G2 P3 Ab0" mean? A. The woman has delivered 3 children, 2 of whom are living; her blood type in Ab0 B. The woman has been pregnant twice with 3 children (twins and another child), and all her children are living C. The woman has been pregnant 3 times, has delivered 2 children, and has had no abortions D. The woman has been pregnant 3 times, has 2 living children, and has had no spontaneous abortions

B

What is the major cause of decreased saliva production in older adults? A. Decreased fluid intake B. Use of anticholinergic medications C. A diminished sense of taste and smell D. Normal aging process

B

What is the most common site of breast tumors? A. upper inner quadrant B. upper outer quadrant C. lower inner quadrant D. lower outer quadrant

B

What is the yin/yang theory of health? A. Health exists in the absence of illness. B. Health exists when all aspects of the person are in perfect balance. C. Health exists when physical, psychological, spiritual, and social needs are met. D. Health exists when there is optimal functioning.

B

What occurs during transduction (the first phase of nociceptive pain)? A. Chemical mediators are neutralized to decrease the perception of pain. B. Pain signals move from the site of origin to the spinal cord. C. The brain interprets the pain signal. D. The pain impulse moves from the spinal cord to the brain.

B

What problems are associated with smoking and the use of oral contraceptives A. Increased risk for alcoholism and cirrhosis of the liver B. Thrombophlebitis and pulmonary embolism C. Infertility and weight gain D. Urinary tract infections and skin cancer

B

What statement is an example of a patient with dysarthria? A. When asked a question, the patient responds fluently but uses words incorrectly or makes up words so that speech may be incomprehensible B. The word choice and grammar are appropriate, but the sounds are distorted so speech is unintelligible C. The pitch and volume of the words are difficult and voice may be hoarse, but language is intact D. Comprehension is intact but there is difficulty in expressing thoughts, with nouns and verbs being the dominant word choice

B

What type of database is most appropriate for an individual who is admitted to a long-term care facility? A. Focused B. Complete C. Emergency D. Follow-up

B

When an otoscopic examination is performed on an older adult patient, the tympanic membrane may be: A. thinner than that of a younger adult. B. whiter than that of a younger adult. C. pinker than that of a younger adult. D. more mobile than that of a younger adult.

B

When assessing a patient's ear with an otoscope, the patient's head should be positioned: A. Tilted toward the examiner B. Tilted away from the examiner C. As vertical as possible D. Tilted down

B

When assessing an infant, the examiner completes the Ortolani maneuver by: A. Lifting the newborn and noting a C-shaped curvature of the spine B. Gently lifting and abducting the infant's flexed knees while palpating the greater trochanter with the fingers C. Comparing the height of the tops of the knees when the knees are flexed up D. Palpating the length of the clavicles

B

When assessing for sound using percussion, the nurse will use A Direct percussion B. Indirect percussion

B

When assessing for the presence of a herniated nucleus pulposus, the examiner would: A. instruct the patient to do a knee bend. B. raise each of the patient's legs straight while keeping the knee extended. C. abduct and adduct the patient's legs while keeping the knee extended. D. dont raise each of the patient's legs straight while keeping the knee extended.

B

When assessing range of motion, which of the following would be important for the nurse to do to help the patient understand what the nurse is asking? A. Anchor the joint with one hand while other hand slowly moves the joint to its limit. B. Model the movements C. Palpate each joint. D. Familiarize himself or herself with type of each joint and normal range of motion

B

Which of the following pairs of sinuses is absent at birth, is fairly well developed between 7 and 8 years of age, and is fully developed after puberty? A. Ethmoid B. Frontal C. Maxillary D. Sphenoid

B

When interviewing a patient who does not speak English, the examiner should: A. Take advantage of family members who are readily available and willing to assist B. Use a qualified medical interpreter who is culturally literate C. Seek as much information as possible and then continue with the physical examination D. Wait until a qualified medical interpreter is available before starting the interview

B

When performing percussion, the examiner A. taps fingertips over bony processes. B. strikes the stationary finger at the distal interphalangeal joint. C. strikes the flank area with the palm of the hand. D. strikes the stationary finger at the proximal interphalangeal joint.

B

When preparing the physical setting for an interview, the interviewer should A. stand next to the patient to convey a professional demeanor. B. conduct the interview at eye level and at a distance of 4 to 5 feet. C. reduce noise by turning the volume on the television or radio down. D. set the room temperature between 64° F and 66° F.

B

When recording information for the review of systems, the interviewer must document A. "negative" under the system heading. B. the presence or absence of all symptoms under the system heading. C. objective data that support the history of present illness. D. physical findings, such as skin appearance, to support historic data.

B

When taking the health history for a child, what information, in addition to that for an adult, is usually obtained? A. Coping and stress management B. A review of immunizations received C. Environmental hazards D. Hospitalization history

B

When the nurse is evaluating the reliability of a patient's responses, which of these statements would be correct? The patient: A. Has a history of drug abuse and therefore is not reliable. B. Provided consistent information and therefore is reliable. C. Smiled throughout interview and therefore is assumed reliable. D. Would not answer questions concerning stress and therefore is not reliable.

B

Which demonstrates a good understanding of the interview process? A. The nurse stops the patient each time something is said that is not understood B. The nurse spends more time listening to the patient than talking C. The nurse is consistently thinking of his or her next response so the patient will know her or she is understood D. The nurse uses "why" questions to seek clarification of unusual symptoms or behaviors

B

Which health belief practice is associated with patients who are of American Indian heritage? A. Wearing bangle bracelets to ward off evil spirits B. Believing that forces of nature must be kept in natural balance C. Using swamp root as a traditional home remedy D. Believing in a shaman as a traditional healer

B

Which is a structure that secretes a thin, milky alkaline fluid to enhance the viability of sperm? A. Cowper gland B. Prostate gland C. Median sulcus D. Bulbourethral gland

B

Which is the first physical change associated with puberty in girls? A. Areolar elevation B. Breast bud development C. Height spurt D. Pubic hair development

B

Which of the following best describes the purpose for a general survey? A. To provide the information necessary for the diagnosis. B. Information obtained will help to guide the physical assessment C. This time provides an opportunity for the patient to relax D. To provide the vital signs prior to starting the exam

B

Which of the following children is at risk for recurrent otitis media (OM)? A. A 2-year-old child who has had two ear infections in the past 6 months B. An 18-month-old infant who has had three episodes of ear infections in a 5-month period C. An 18-month-old infant who lives with a smoker D. A 6-month-old infant who has a sibling who had tubes inserted at 3 years of age

B

Which of the following documentation statements indicates a normal assessment of the perianal area? A. "Small round opening in the anal area." B. "Anus moist, color darker than adjacent tissues. No lesions or discharge. Opening tightly closed." C. "Anal area dark pink, moist, with 0.5-cm shiny blue skin sac at 5 o'clock." D. "Anus with flabby skin sac at 7 o'clock."

B

Which of the following groups of individuals need to be tested for the presence of color blindness (deficiency)? A. White girls between the ages of 4 and 8 years B. White boys between the ages of 4 and 8 years C. Asian girls between the ages of 3 and 6 years D. Black boys between the ages of 10 and 15 years

B

Which of the following is a clinical manifestation in a patient with chronic obstructive pulmonary disease (COPD)? A. Periodic breathing patterns B. Prolonged expiration C. Unequal chest expansion D. Hyperventilation

B

Which of the following is a normal range for a patient's temperature measured using an oral thermometer? A. 37.5° C to 39.2° C B. 35.8° C to 37.3° C C. 36.2° C to 38.2° C D. 34.0° C to 34.9° C

B

Which of the following is an example of subjective data? A. Blood glucose 126 md/dL B. Pain rated at 7 out of 10 C. Heart rate of 76 bpm D. Bruising on lower leg

B

Which of the following is the most reliable indicator for chronic pain? A. Blood drug levels B. Patient self-report C. Magnetic resonance imaging (MRI) results D. Tissue enzyme levels

B

Which of the following statements r/t aggravating symptoms or triggers of headaches is true? A. Poor posture may trigger a migraine headache B. Alcohol consumption may precipitate the onset of cluster or migraine headaches C. Premenstrual hormonal fluctuations may precipitate the onset of cluster headaches D. Certain foods such as chocolate or cheese may precipitate the onset of tension headaches

B

Which of the following statements regarding cultural/racial differences in the treatment of pain is true? A. Neurotransmitters are more concentrated in white individuals than in black and Hispanic individuals. B. White individuals receive more analgesic therapy than black or Hispanic individuals with similar symptoms. C. Pain modulation is more highly developed in black and Hispanic individuals. D. Black and Hispanic individuals have been found to have a higher pain tolerance than white individuals.

B

Which of the following would be most appropriate when weighing an infant? A. If the infant can sit up, he/she can be weighed on a standard upright scale. B. The infant should be weighed undressed on a platform-type scale. C. It's okay to weigh the infant fully dressed. D. Weigh the mother then have her hold the infant and subtract the mother's weight from the result.

B

Which of these is the correct technique to assess patency of the anal sphincter? A. Inspect the anus and ask the patient to strain B. Palpate the anus with a gloved finger to elicit sphincter control C. Percuss above the anal canal for dull resonance D. There are no sensory nerves in the anal canal or rectum

B

Which population has the highest incidence of benign prostatic hypertrophy (BPH)? A. European Americans B. African Americans C. Hispanics D. Asians

B

Which risk factor increases the risk for skin disease and breakdown among older adults? A. Decreased vascular fragility B. A lifetime of environmental trauma C. Increased thickness of the skin D. Loss of protective cushioning of the dermal skin layer

B

Which statement about the apices of the lungs is true? The apices of the lungs: A. Are at the level of the second rib anteriorly. B. Extend 3 to 4 cm above the inner third of the clavicles. C. Are located at the sixth rib anteriorly and the eighth rib laterally. D. Rest on the diaphragm at the fifth intercostal space in the midclavicular line (MCL).

B

Which statement best illustrates the difference between religion and spirituality? A. Religion reflects and individual's reaction to life events whereas spirituality is based on whether the individual attends religious services B. Religion is characterized by identification of a higher being shaping one's destiny, whereas spirituality reflects an individual's perception of one's life having worth or meaning C. Religion is the expression of spiritual awakening whereas spirituality is based on belief in divine right D. Religion is the active interpretation of one's spirituality

B

Which statement concerning the anal canal is true? The anal canal: A. Is approximately 2 cm long in the adult. B. Is the outlet for the gastrointestinal tract. C. Contains hair and sebaceous glands. D. Slants backward toward the sacrum.

B

Which statement is true in regard to pain? A. Nurses' attitudes toward their patients' pain are unrelated to their own experiences with pain B. The cultural background of a patient is important in a nurse's assessment of that patient's pain C. A nurse's area of clinical practice is more likely to determine his or her assessment of a patient's pain D. A nurse's years of clinical experience and current position are a strong indicator of his or her response to patient pain

B

Which technique is the second step in the sequence for abdominal assessment? A. Inspection B. Auscultation C. Percussion D. Palpation

B

Which technique would the nurse use to noninvasively assess arterial oxygen saturation? A. Arterial blood gas B. Pulse oximeter C. Blood Pressure D. Respiratory rate

B

Which theory has been expanded in an attempt to study the degree to which a person's lifestyle reflects his or her traditional heritage? A. Congruence mechanism B. Heritage consistency C. Behavior theory D. Socialization experience

B

While evaluating the health history, the nurse determines that the patient subscribes to the hot/cold theory of health. Which of the following would most likely describe this patient's view of wellness? A. The phlegm will be replaced with dryness. B. The humors must be balanced. C. Good is hot. D. Evil is hot.

B

While examining the broken arm of a 4-year-old boy, select the appropriate assessment tool to evaluate his pain status A. 0 to 10 numeric rating scale B. Wong-Baker scale C. Simple descriptor scale D. 0 to 5 numeric rating scale

B

While taking a history, the patient describes a burning, painful sensation that moves around the toes and bottoms of the feet. These symptoms suggest: A. Nociceptive pain B. Neuropathic pain C. Visceral pain D. Muscular pain

B

Why is it important for the nurse to use a routine examination sequence? A. The nurse doesn't need to have a routine sequence. B. It helps to minimize the number of position changes for patient and the nurse. C. The nurse needs to have all necessary equipment prepared in advance. D. It establishes trust with the patient.

B

With older adults, how should the examiner proceed with the interview? A. Proceed in a more organized an concise manner B. Consider the fatigue of the older person and break the interview into shorter segments C. Ask a family member to complete some of the records while moving ahead with the interview D. Raise your voice if the patient does not appear to hear you

B

You are assessing a patient's tympanic membrane and suspect an infection of acute purulent otitis media. Which of the following findings supports this? A. Absent light reflex, bluish drum, oval dark areas B. Absent light reflex, reddened drum, bulging drum C. Oval dark areas on drum D. Absent light reflex, air-fluid level, or bubbles behind drum

B

A patient has a urinary catheter. Which assessment should be done each time vital signs are taken on the patient? A. Adding additional water into the system to ensure that the balloon is inflated B. Emptying the drainage bag and noting the output C. Observing the color of the output D. Repositioning the catheter to ensure there is no occlusion or blockage

C

You are auscultating breath sounds on a patient. Which of the following best describes how to proceed? A. Hold the bell of the stethoscope against the chest wall; listen to the entire right field and then the entire left field B. Hold the diaphragm of the stethoscope against the chest wall; listen to one full respiration in each location, being sure to do side-to-side comparisons C. Listen from the apices to the bases of each lung field using the bell of the stethoscope D. Select the bell or diaphragm depending on the quality of sounds heard; listen for one respiration in each location, moving from side to side

B

You assess a patient for arterial deficit in the lower extremities. After raising the legs 12 inches off the table and then having the person sit up and dangle the leg, the color should return in: A. 5 seconds or less B. 10 seconds or less C. 15 seconds D. 30 seconds

B

You assess a patient who reports a cough. The characteristic timing of the cough of chronic bronchitis is described as: A. Continuous throughout the day. B. Productive cough for at least 3 months of the year for 2 consecutive years C. Occurring in the afternoon or evening because of exposure to irritants at work D. Occurring in the early morning

B

You conduct an admission interview. Because you are expecting a phone call, you stand near the door. Which would be a more appropriate approach? A. Arrange to have someone page you so you can sit on the side of the bed B. Have someone else answer the phone so you can give the patient your full attention C. Use this approach given the circumstances D. Arrange for a time free of interruptions after the initial physcial examination is complete

B

You palpate the maternal abdomen at approximately 35 weeks. Your left hand is on the maternal right, and your right hand is on the maternal left. What maneuver is this? A. Leopold's first maneuver B. Leopold's second maneuver C. Schmidt's third maneuver D. Schmidt's second maneuver

B

You suspect an infant's head is of abnormal size and can use which procedure to verify these findings? A. Palpation B. Measuring tape C. Observing for symmetry of facial features D. Noting absence of the tonic neck reflex

B

You will hear a split S2 most clearly in which area? A. Apical B. Pulmonic C. Tricuspid D. Aortic

B

herpes zoster infection (shingles) is characterized by: A. A bacterial cause B. Lesion on only one side of body; does not cross midline C. Absence of pain or edema D. Pustular, umbilicated lesions

B

Which of the following actions/behaviors in the critical-thinking process are important for the novice nurse to remember? (Select all that apply.) A. Disregard initial cues B. Approach assessment with a nonjudgmental attitude C. Cluster associated assessment data D. Perform assessment in whatever manner works for you. E. Avoid making assumptions

B, C, & E

Which of the following questions would be appropriate for the nurse to ask when a patient complains of diarrhea? (Select all that apply.) A. "Have you ever had clay-colored stools?" B. "Have you traveled recently?" C. "Do you eat breakfast?" D. "Have you eaten at a restaurant in the past few days?" E. Have you recently been prescribed antibiotics?"

B, D & E

A patient has blurred peripheral vision. You suspect glaucoma and assess the visual fields. A person with normal vision would see your moving finger temporally at: A. 50 degrees B. 60 degrees C. 90 degrees D. 180 degrees

C

A 30-year-old woman reports having persistent intense pain in her right arm related to trauma sustained from a car accident 5 months ago. She states that the slightest touch or clothing can exacerbate the pain. The report is suggestive of: A. Referred pain B. Psychogenic pain C. Complex regional pain I D. Cutaneous pain

C

A 65-year-old patient is experiencing pain in his left calf when he exercises that disappears after resting for a few minutes. The nurse recognizes that this description is most consistent with _______ the left leg. A. Venous obstruction of B. Claudication due to venous abnormalities in C. Ischemia caused by a partial blockage of an artery supplying D. Ischemia caused by the complete blockage of an artery supplying

C

A 70-year-old woman has come for a health examination. Which of the following is a common age-related change in the curvature of the spinal column? A. Lordosis B. Scoliosis C. Kyphosis D. Lateral scoliosis

C

A 74-year-old man has come for a health examination. A normal age-related change in the scrotum would be: A. Testicular atrophy B. Testicular hypertrophy C. Pendulous scrotum D. Increase in scrotal ruage

C

A 92-year-old patient has had a stroke. The right side of his face is drooping. The nurse might also suspect which of these assessment findings? A. Epistaxis B. Rhinorrhea C. Dysphagia D. Xerostomia

C

A bimanual technique may be the preferred approach for a woman: A. Who is pregnant B. Who is having the first breast examination by a health care provider C. With pendulous breasts D. Who has felt a change in the breast during self-examination

C

A bruit heard while auscultating the carotid artery of a 65-year-old patient is caused by A. increased viscosity of blood. B. rapid blood flow through the carotid artery. C. turbulent blood flow through the carotid artery. D. decreased velocity of blood flow through the carotid artery.

C

A configuration of individual lesions arranged in circles or arcs, as occurs with ringworm, is described as a: A. Linear lesion B. Clustered lesion C. Annular lesion D. Gyrate lesion

C

A deep recess formed by the peritoneum between the rectum and the cervix is called A. a cystocele. B. the Chadwick sign. C. a rectouterine pouch. D. a rectocele.

C

A flat discoloration of the skin that is less than 1 cm in diameter is termed A. Cancer B. Papule C. Macule D. Bulla

C

A genogram is used for which reason? A. Past history B. Past health history, specifically hospitalizations C. Family history D. The 8 characteristics of presenting symptoms

C

A murmur is heard after S1 and before S2. This murmur would be classified as: A. Diastolic (possibly benign) B. Diastolic (always pathologic) C. Systolic (possibly benign) D. Systolic (always pathologic)

C

A nurse is taking complete health histories on all of the patients attending a wellness workshop. On the history form, one of the written questions asks, "You don't smoke, drink, or take drugs, do you?" This question is an example of: A. Talking too much. B. Using confrontation. C. Using biased or leading questions. D. Using blunt language to deal with distasteful topics.

C

A nurse precepting a student nurse asks, "What's the most important step in the critical-thinking process?" A. Clustering subjective and objective data B. Analyzing health data C. Using evidence-based assessment techniques D. Prioritizing health concerns

C

A patient asks the nurse, "May I ask you a question?" This is an example of: A. An open-ended question B. A reflective question C. A closed questions D. A double barreled question

C

A patient has recently received health insurance and would like to know how often he should visit the provider. How do you respond? A. "It would be most efficient if you visit on an annual basis." B. There is no recommendation for the frequency of healthcare visits." C. "Your visits may vary, depending on your level of wellness." D. "Your visits will be based on your preference."

C

A patient has severe bilateral lower extremity edema. The most likely cause is A. Raynaud phenomenon. B. an aortic aneurysm. C. heart failure. D. an infection of the right great toe.

C

A patient in whom a seizure disorder was recently diagnosed plans to continue a career as a pilot. At this time in the interview, the nurse begins to question the patient's A. thought process. B. intellect. C. judgment. D. perception.

C

A patient is being seen in the clinic for complaints of "fainting episodes that started last week." How should you proceed with the examination? A. Take the blood pressure in both arms and thighs B. Ask the person to walk a few paces and then take the blood pressure C. Record the blood pressure in the lying, sitting, and standing positions D. Record the blood pressure in the lying and sitting positions and average these number to obtain a mean blood pressure

C

A patient says she is very nervous and nauseated, and she feels as if she will vomit. This data would be what type of data? A. Objective B. Reflective C. Subjective D. Introspective

C

A patient states that he is frequently constipated, and when he has a bowel movement he has rectal bleeding and pain. He does not feel any mass at his anal opening. "Do I have hemorrhoids, or is there something else wrong with me?" The examiner completes a rectal examination and explains that: A. There is an indication of rectal prolapse B. It appears to be a pilonidal cyst C. The symptoms are consistent with internal hemorroids D. The problem is probably encopresis

C

A pleural friction rub is best detected by: A. Observation B. Palpation C. Auscultation D. Percussion

C

A positive Phalen test and Tinel sign are found in a patient with: A. A torn meniscus B. Hallux valgus C. Carpal tunnel syndrome D. Tennis elbow

C

A risk factor for melanoma is: A. Brown eyes B. Darkly pigmented skin C. Skin that freckles or burns before tanning D. Use of sunscreen products

C

A scooped-out, shallow depression in the skin is called a(n) A. excoriation. B. ulcer. C. erosion. D. fissure.

C

A slight protrusion of the eyeballs may be noticed when examining individuals who come from which ethnic/cultural group? A. Hispanic B. American Indian C. African American D. Asian

C

A throbbing, unilateral pain associated with nausea, vomiting, and photophobia is characteristic of: A. Cluster headache B. Subarachnoid hemorrhage C. Migraine headache D. Tension headache

C

A water-hammer or Corrigan pulse is associated with A. conduction disturbance. B. hyperkinetic states. C. aortic valve regurgitation. D. decreased cardiac output.

C

A woman reports nausea, fatigue, breast tenderness, urinary frequency, and amenorrhea. These are: A. Probable signs of pregnancy B. Positive sings of pregnancy C. Presumptive signs of pregnancy D. Signs of stress

C

Although a full mental status examination may not be required for every patient, the health care provider must address the four main components during a health history and physical examination. The four components are A. language, orientation, attention, and abstract reasoning. B. mood, affect, consciousness, and orientation. C. appearance, behavior, cognition, and thought processes. D. memory, attention, thought content, and perceptions.

C

An area of thin shiny skin with decreased visibility of normal skin markings is most likely: A. Lichenification B. Plaque C. Atrophy D. Keloid

C

An example of objective data is A. a report of impaired mobility from left knee pain as evidenced by an inability to walk, swelling, and pain on passive range of motion. B. a complaint of left knee pain. C. crepitation in the left knee joint. D. left knee has been swollen and hot for the past 3 days.

C

An older woman is having an annual mammogram. Before the mammogram, the nurse does a breast examination. Expected normal findings would include A. increased glandularity. B. a unilateral venous pattern. C. palpable, firm, stringy lactiferous ducts. D. yellow colostrum expressed from the nipple.

C

Approximately 2 to 3 weeks before labor, the woman will experience which sign that the baby has "dropped" into the pelvis? A. Extreme fatigue B. Braxton-Hicks contractions C. Lightening D. Back pain

C

At the end of the examination, the examiner should A. compare objective and subjective data for discrepancies. B. have findings confirmed by another provider. C. review the findings with the patient. D. complete documentation before leaving the examination room.

C

At which phase does the individual become aware of a painful sensation? A. Modulation B. Transduction C. Perception D. Transmission

C

Before withdrawing the speculum, the examiner swabs the cervix with a swab soaked in acetic acid. This examination is done to assess for: A. Herpes simplex virus B. Contact dermatitis C. Human papillomavirus D. Carcinoma

C

Cardiac output in a pregnant women: A. Drops dramatically B. Remains the same C. Increases along with stroke volume D. Decreases along with stroke volume

C

Ms T has come for a prenatal visit. She complains of dependent edema, varicosities in the legs, and hemorrhoids. What is the best response? A. "If these symptoms persist, we will perform an amniocentesis." B. "If these symptoms persist, we will discuss having you hospitalized." C. "The symptoms are caused by the pressure of the growing uterus on the veins. They are usual conditions of pregnancy." D. "At this time, the symptoms are a minor inconvenience. You should learn to accept them."

C

Narrow palpebral fissures, epicanthal folds, and midfacial hypoplasia are characteristic of: A. Chronic childhood allergies B. Congenital hypothyroidism C. Fetal alcohol syndrome D. Down syndrome

C

Normal age-related findings in the lower extremities of an 80-year-old woman would be: A. Crepitus B. Joint swelling C. Diminished strength bilaterally D. Unilateral muscle atrophy

C

Normal assessment findings of an infant's fontanel include A A molded form B. Normal, round shape C. Even with the skull D. Palpable suture lines

C

On a patient's second day in an acute care hospital, the patient complains about the "bugs" on the bed. The bed is clean. This would be an example of altered A. Thought processes B. Orientation C. Perception D. Higher intellectual function

C

On examination, the newborn's fontanels should feel: A. Tense or bulging B. Depressed or sunken C. Firm, slightly concave, and well defined D. Pulsating

C

On examining a patient's nails, you note that the angle of the nail base is >160 degrees and that the nail base feels spongy to palpation. These findings are consistent with: A. Actue respiratory distress syndrome B. Normal findings for the nails C. Congenital heart disease and COPD D. Atelectasis

C

Orchitis is (are) A. hard, subcutaneous plaques associated with painful bending of the erect penis. B. a circumscribed collection of serous fluid in the tunica vaginalis surrounding the testes. C. an acute inflammation of the testes. D. a meatus opening on the dorsal side of the glans or shaft.

C

PQRSTU is a mnemonic that helps the clinician to remember to address characteristics specific to A. the ability to perform activities of daily living (ADLs). B. substance use and abuse. C. pain presentation. D. severity of dementia.

C

Patients who have Parkinson disease usually have which of the following characteristic styles of speech? A. A garbled manner B. Loud, urgent C. Slow, monotonous D. Word confusion

C

Prostatic hypertrophy occurs frequently in older men. The symptoms that my indicate this problem are: A. Polyuria and urgency B. Dysuria and oliguria C. Straining, loss of force, and sense of residual urine D. Foul-smelling urine and dysuria

C

Select the best description of the anal canal. A. 12-cm-long portion of the large intestine B. Involuntary control of the parasympathetic nervous system C. 3.8-cm-long outlet of the gastrointestinal tract D. S-shaped portion of the colon

C

Select the sequence of techniques used during an examination of the abdomen. A. Percussion, inspection, palpation, auscultation B. Inspection, palpation, percussion, auscultation C. Inspection, auscultation, percussion, palpation D. Auscultation, inspection, palpation, percussion

C

Soft, pointed, fleshy papules that occur on the genitalia caused by human papillomavirus (HPV) are known as A. chancres. B. urethritis. C. genital warts. D. varicoceles.

C

Subjective data is gathered from the ________. A. Physical exam B. Lab work only C. Interview D. General survey

C

Tenderness during abdominal palpation is expected when palpating the: A. Liver edge B. Spleen C. Sigmoid colon D. Kidneys

C

The "A" in the ABCDEF rule for skin cancer stands for: A. Accuracy B. Appearance C. Asymmetry D. Attenuated

C

The 6 eye muscles that control eye movement are innervated by cranial nerves: A. II, III, V B. IV, VI, VII C. III, IV, VI D. II, III, VI

C

The _____________ coordinates movement, maintains equilibrium, and helps maintain posture. A. basal ganglia B. extrapyramidal system C. cerebellum D. upper and lower motor neurons

C

The absence of bowel sounds is established after listening for: A. 1 full minute B. 3 full minutes C. 5 full minutes D. None of the above

C

The concept of health and healing has evolved in recent years. Which is the best description of health? A. Health is the absence of disease. B. Health is a dynamic process toward optimal functioning. C. Health depends on an interaction of mind, body, and spirit within the environment. D. Health is the prevention of disease.

C

The cover test is used to assess for: A. Nystagmus B. Peripheral vision C. Muscle weakness D. Visual acuity

C

The examiner is going to inspect and palpate for a hernia. During this examination, the man is instructed to: A. Hold his breath during palpation B. Cough after the examiner has gently inserted the examination finger into the rectum C. Bear down when the examiner's finger is at the inguinal canal D. Relax in supine position while the examination finger is inserted into the canal

C

The examiner measures a patient's legs for length discrepancy. Which is a normal finding? A. No difference in measurements B. 0.5cm difference C. Within 1cm of each other D. 2cm difference

C

The examiner notes small, round, white, shiny papules on the hard palate and gums of a 2-month-old infant. What is the significance of this finding? A. These are aphthous areas or ulcers that are the result of sucking B. Teeth buds are beginning to appear C. This is a normal finding called Epstein pearls D. It indicates the presence of a monilial infection

C

The examiner should auscultate for carotid bruits if the patient A. is pregnant and has gestational diabetes. B. has enlarged, tender cervical lymph nodes. C. is middle aged or older. D. complains of abdominal pain.

C

The function of the pulmonic valve is to: A. Divide the left atrium and left ventricle B. Guard the opening between the right atrium and right ventricle C. Protect the orifice between the right ventricle and the pulmonary artery D. Guard the entrance to the aorta from the left ventricle

C

The general survey consists of four distinct areas. These areas include A. gait, range of motion, mental status, and behavior. B. level of consciousness, personal hygiene, mental status, and physical condition. C. physical appearance, body structure, mobility, and behavior. D. mental status, speech, behavior, and mood and affect.

C

The jugular venous pressure is an indirect reflection of the A. cardiac cycle. B. synchronization of mechanical activity. C. heart's efficiency as a pump. D. conduction effectiveness.

C

The labyrinth of the inner ear is responsible for maintaining the body's: A. Air conduction. B. Binaural interaction. C. Equilibrium. D. Pressure equalization.

C

The lens of the eye functions as a: A. Mediator of light. B. Sensory facilitator. C. Refracting medium. D. Controller of intraocular pressure.

C

The location in the brain where optic nerve fibers from the temporal fields of vision cross over is identified as the: A. Fovea centralis. B. Choroid. C. Optic chiasm. D. Optic disc.

C

The major factor contributing to the need for cultural care nursing is: A. An increasing birth rate B. Limited access to health care services C. Demographic change D. A decreasing rate of immigration

C

The nurse is assessing range of motion and notes a grating sound when examining the shoulder joint. The nurse would correctly document which of the following choices? A. Limited range of motion of the shoulder joint B. Shoulder deformity C. Crepitus of the shoulder joint D. Shoulder atrophy

C

The nurse is assessing the eyes of a client and notes a drooping of the left eyelid. The nurse would correctly chart which of the following conditions? A. nystagmus B. strabismus C. ptosis D. diplopia

C

The nurse is assessing the fundal height of a pregnant client and notes the fundus is at the level of the umbilicus. The nurse would correctly estimate the weeks in pregnancy as which of the following? A. 10 to 12 B. 16 C. 20 to 22 D. 38

C

The nurse is calling the health care provider about a patient's changing condition. Which of the following would be included in the SBAR communication? A. Summary, better plan, accurate diagnosis, and rights B. Situation, background, all vitals, and review of orders C. Situation, background, assessment, and recommendation D. Subjective information, background, assessment, and revisions needed

C

The nurse is completing an assessment of a nonverbal patient who had an appendectomy the previous day. The patient is restless, holding his hand over his abdomen, sweating, and his vital signs are: heart rate 100 bpm, respirations 20, blood pressure 135/90. Which of the following would be the most appropriate action by the nurse? A. Teach the patient the importance about getting out of bed and walking after surgery. B. Offer to turn on healing music. C. Ask the patient to nod "yes" or "no" and ask him or her if he or she is experiencing pain. D. Chart vital signs and assessment and determine normal findings.

C

The stethoscope bell should be pressed lightly against the skin so that: A. Chest hair doesn't simulate crackles B. High-pitched sounds can be heard better C. The bell does not act as a diaphragm D. The bell does not interfere with amplification of heart sounds

C

The nurse is describing a weak, thready pulse on the documentation flow sheet. Which statement is correct? A. "Is easily palpable; pounds under the fingertips." B. "Has greater than normal force, then suddenly collapses." C. "Is hard to palpate, may fade in and out, and is easily obliterated by pressure." D. "Rhythm is regular, but force varies with alternating beats of large and small amplitude."

C

The nurse is examining an adult male and notes thick and curly hair over the pubis area, a pear shaped scrotum, and slightly darkened skin on the penis. The nurse would correctly choose which of the following actions? A. Ask the client about risk taking sexual practices. B. Notify the physician of the findings. C. Document the findings as normal. D. Ask the client about childhood illnesses.

C

The nurse is examining only the rectal area of a woman and should place the woman in what position? A. Lithotomy B. Prone C. Left lateral decubitus D. Bending over the table while standing

C

The nurse is examining the genitalia of a male child and notes that the urinary meatus appears on the underside of the glans penis. The nurse would correctly document this finding as which of the following? A. Normal B. Epispadias C. Hypospadias D. Paraphimosis

C

The nurse is examining the genitalia of a male client. On palpation of the right inguinal area, the nurse detects a bulge as the client bears down. The nurse would correctly interpret this finding as which of the following conditions? A. Prostatitis B. Cremasteric reflex C. Hernia D. Variocele

C

The nurse is inspective the spine of a client and notes the presence of an exaggerated lumbar curve. The nurse would correctly document which of the following choices? A. Spinal list B. Kyphosis C. Lordosis D. Flattened lumbar curve

C

The nurse is performing a neurological assessment on a client experiencing vertigo. The nurse wants to perform the Romberg test. The nurse would correctly provide which set of instructions to the client? A. "Walk across the room by placing one foot in front of the other heel to toes." B. "Walk on your toes, then on your heels." C. "Stand with your feet together, arms at sides, and eyes open." D. "Touch your finger to your nose alternating hands."

C

The nurse is performing a vaginal exam on a female client and palpates a protrusion into the posterior vaginal wall. The nurse would suspect which of the following conditions in this situation? Bartholin's gland infection Cystocele Correct! Rectocele Ovarian cyst

C

The nurse is performing reflex testing on a client and uses the reflex hammer to gently strike the forearm about 5 cm (2 inches) above the wrist. The nurse is assessing which of the following reflexes? A. Biceps B. Triceps C. Brachioradialis D. Patellar

C

The nurse is preparing to teach a woman about BSE. Which statement by the nurse is correct? A. "BSE is more important than ever for you because you have never had any children." B. "BSE is so important because one out of nine women will develop breast cancer in her lifetime." C. "BSE on a monthly basis will help you become familiar with your own breasts and feel their normal variations." D. "BSE will save your life because you are likely to find a cancerous lump between mammograms."

C

The nurse is presenting a class to young adults about the risks of smoking and chewing tobacco. One of the participants asks what physical signs would be present if one developed cancer of the mouth related to chewing tobacco. The nurse would respond by stating the physical findings include A. Pallor of the oral mucosa B. Excessive salivation C. Ulcer on lower lip or tongue D. Tender lymph nodes of the neck

C

The nurse is reviewing the blood supply to the arm. The major artery supplying the arm is the _______ artery. A. ulnar B. radial C. brachial D. deep palmar

C

The nurse palpates the abdominal aorta of an adult client and find that it measures approximately 6 cm in diameter. The next step for the nurse to take is A. To continue to apply deep palpation inferiorly to assess accurate measurement B. To palpate lightly to just under the xiphoid process C. Discontinue palpation and document findings D. Auscultate for bruits

C

The ophthalmoscope has 5 apertures. Which aperture is used to assess the eyes of a patient with undilated pupils? A. Grid B. Slit C. Small D. Large

C

The precordium is: A. a synonym for the mediastinum. B. the area on the chest where the apical impulse is felt. C. the area on the anterior chest overlying the heart and great vessels. D. a synonym for the area where the superior and inferior venae cavae return unoxygenated venous blood to the right side of the heart.

C

The presence of primitive reflexes in a newborn infant is indicative of: A. prematurity of the infant B. mental retardation C. immaturity of the nervous system D. spinal cord alterations.

C

The sequence of an examination changes from beginning with the thorax to that of head to toe when the patient is in what age category? A. The infant B. The preschool child C. The school-age child D. The adolescent

C

The statement "Reason for seeking care" has replaced the "chief complaint." This change is significant because: A. The "chief complaint" is really a diagnostic statement B. The newer term allows another individual to supply the necessary information C. The newer term incorporated wellness needs D. The "reason for seeking care" can incorporate the history of the present illness

C

The symptoms occurring with lactose intolerance include: A. hematemesis B. gray stools C. bloating and flatulence D. anorexia

C

What term is used to describe slow, twisting muscle movements that resemble a snake or worm? A. Ataxia B. Vestibular function C. Athetosis D. Flaccid

C

When assessing hearing acuity in a 6-month-old child, the examiner would: A. Use an audiometer B. Observe for shyness and withdrawal C. Watch for head turning when saying the child's name D. Test the startle (Moro) reflex

C

When auscultating lung sounds, it is important for the nurse to do which of the following? A. Anterior auscultation should include 9 spots on each side. B. Quickly move through the assessment to decrease risk for fainting or hyperventilation. C. Tell the patient to stop the assessment if he/she begins to feel dizzy. D. Complete assessment of right lung fields then move to auscultate the left lung fields.

C

When evaluating day-to-day oxygenation changes in the skin color of a client with dark skin, you would do which of the following things? A. Realize you cannot tell changes in dark skin and chart "not applicable" B. Use a bright lamp and a magnifying glass on the client's skin C. Inspect the client's lips, oral mucosa, sclera, conjunctivae, and palms D. Inspect the skin color the same way you would inspect lighter skin

C

When examining a patient's eyes, the nurse recalls that stimulation of the sympathetic branch of the autonomic nervous system: A. causes pupillary constriction B. adjusts the eye for near vision C. elevates the eyelid and dilates the pupil D. causes constriction of the ciliary body

C

When examining for tactile fremitus, it is important to: A. Ask the patient to breathe quickly B. Ask the patient to cough C. Palpate the chest symmetrically D. Use the bell of the stethoscope

C

When performing a respiratory assessment on a patient, the nurse notices a costal angle of approximately 90 degrees. This characteristic is: A. Observed in patients with kyphosis. B. Indicative of pectus excavatum. C. A normal finding in a healthy adult. D. An expected finding in a patient with a barrel chest.

C

When performing indirect percussion, the stationary finger is struck: A. At the ulnar surface B. At the middle joint C. At the distal interphalangeal joint D. Wherever it is in contact with the skin

C

When taking the health history, the patient complains of pruritus. What is a common cause of this symptom? A. Hyperpigmentation B. Melasma C. Drug reactions D. Excessive bruising

C

When teaching the breast self-examination, you would inform the woman that the best time to conduct breast self-examination is: A. at the onset of the menstrual period. B. on the 14th day of the menstrual cycle. C. on the 4th to 7th day of the cycle. D. just before the menstrual period.

C

When the examiner enters the examining room, the infant patient is asleep. Which assessment should the examiner perform next? A. Height and weight B. Blood pressure C. Heart, lung, and abdomen D. Temperature

C

Which woman should NOT be referred to a provider for further evaluation? A. A 26-year-old with multiple nodules palpated in each breast B. A 48-year-old who has a 6-month history of reddened and sore left nipple and areolar area C. A 25-year-old with asymmetric breasts and inversion of nipples since adolescence D. A 64-year-old with ulcerated area at tip of right nipple; no masses, tenderness, or lymph nodes palpated

C

Which changes in head circumference measurements in relation to chest measurements will occur from infancy through early childhood? A. A newborn's head should be approximately 5 cm larger than the chest circumference, but by age 2, they should be equal B. The chest grows at a faster rate than the cranium, but at age 1, the measurements will be the same, and after age 2, the chest should be approximately 5 cm larger C. The newborn's head will be approximately 2 cm larger than the chest circumference, but between 6 months and 2 years, they will be about the same D. The head and chest circumferences should be very similar, but between 6 months and 2 years, the chest size will increase and remain that way.

C

Which changes regarding height and weight occur during a person's 80s and 90s? A. Both increase B. Weight increases, and height decreases C. Both decrease D. Both remain the same as during the 70s

C

Which is an appropriate recording of a patients' reason for seeking health care? A. Angina pectoris, duration 2 hours B. Substernal pain radiating to left axilla, 2 hour duration C. "Grabbing" chest pain for 2 hours D. Pleurisy, 2 days duration

C

Which is true regarding the bulbourethral gland? A. It can be palpated during an examination of a female patient B. It can be palpated during an examination of both male and female patients C. It can be palpated during an examination of a male patient D. It cannot be assessed with a rectal examination

C

Which of the following actions by the student nurse is indication that he/she does not know the correct method for assessing the carotid artery? A. Asks patient to take a breath, exhale, and hold briefly while auscultating the carotid artery. B. The patient's neck is in a neutral position. C. He/she is firmly pressing the bell of the stethoscope over the carotid artery. D. Lightly palpates the left carotid artery, releases, and then palpates the right carotid artery.

C

Which of the following are findings r/t testosterone deficiency? A. Increased libido B. Scrotal sac pendulous with less rugae C. Increased fatigue D. Decreased penis size

C

Which of the following can be noted through inspection of a patient's abdomen? A. Fluid waves and abdominal rigidity B. Umbilical eversion and Murphy sign C. Venous pattern, peristaltic waves, and abdominal contour D. Peritoneal irritation, general tympany, and peristaltic waves

C

Which of the following cardiac alterations occurs during pregnancy? A. An increased heart rate and increased blood pressure B. An increased stroke volume and decreased cardiac output C. An increase in cardiac volume and a decrease in blood pressure D. An increase in cardiac output and blood pressure

C

Which of the following has been found to influence pain sensitivity in women? A. Age B. Parity C. Hormonal changes D. Weight

C

Which of the following is an example of objective data? A. A sore throat B. An earache C. Alert and oriented D. Dizziness

C

Which of the following is included in documenting a history source? A. Appearance, dress, and hygiene B. Documented relationship of support systems C. Reliability of informant D. Cognition and literacy level

C

Which of the following organs aid the lymphatic system? A. Liver, lymph nodes and stomach B. Pancreas, small intestine and thymus C. Spleen, tonsils, thymus D. Pancreas, spleen, tonsils

C

Which of the following patients should the nurse assess first? A. A 52-year-old patient with a white blood cell count of 22,000 cells/mm3 B. An 89-year-old patient with a urinary tract infection who is confused C. A 48-year-old patient with shortness of breath and pulse oximeter reading of 88% D. A 59-year-old patient with chest pain that increases with deep inspiration

C

Which of the following statements regarding language barriers and health care is true? A. English proficiency is associated with a lower quality of care. B. Patients with language barriers have a decreased risk for nonadherence to medication regimens. C. Standards have been identified that are important to eliminate health disparities. D. LEP is associated with a higher quality of care.

C

Which of the following statements regarding the results obtained from use of the Snellen chart is true? A. The smaller the denominator, the poorer the vision. B. The smaller the numerator, the poorer the vision. C. The larger the denominator, the poorer the vision. D. The larger the numerator, the better the vision.

C

Which of the following symptoms is greatly influenced by a person's cultural heritage? A. Food intolerance B. Hearing loss C. Pain D. Breast lump

C

Which statement best describes ethnocentrism? A. The government's description of various cultures B. A central belief that accepts all cultures as one's own C. The tendency to view your own way of life as the most desirable D. The tendency to impose your beliefs, values, and patterns of behaviors on a individual from another culture

C

Which statement best reflects the Magicoreligious causation of illness? A. Each being is but a part of a larger structure in the world of nature as it relates to health and illness B. Causality relationship exists, leading to expression of illness C. Belief in the struggle between good and evil is reflected in the regulation of health and illness D. Illness occurs as a result of disturbances between hot and cold reactions

C

Which symptoms suggest benign prostatic hypertrophy? A. Weight loss and bone pain B. Fever, chills, urinary frequency, and urgency C. Difficulty initiating urination and weak stream D. Dark, tarry stools

C

Which two sections of the child's health history become separate sections because of their importance to the child's current health status? A. Play activities and rest patterns B. Prenatal and postnatal status C. Developmental and nutritional history D. Accidents, injuries, and immunizations

C

You are assessing a 75-year-old man. What is an expected finding? A. He will have no decrease in any of his abilities, including response time. B. He will have difficulty on test of remote memory because this typically decreases with age C. It may take him, a little longer to respond but his general knowledge and abilities should not have declined D. He will have had a decrease in his response time due to language loss and a decrease in general knowledge

C

You are assessing a 75-year-old patient's oral cavity. Which of the following would most likely be present? A. Hypertrophy of the gums B. An increased production of saliva C. Decreased ability to identify odors D. Finer and less prominent nasal hair

C

You are assessing a patient's gait. What do you expect to find? A. Gait is varied, depending on the height of the person B. Gait is equal to the length of the arm C. Gait is as wide as the shoulder width D. Gait is half the height of the person

C

You are going to inspect a female patient's breast for retraction. The best position for this part of the examination is: A. Lying supine with arms at the sides B. Leaning forward with hands outstretched C. Sitting with hands pushing into hips D. One arm at the side, the other arm elevated

C

You are performing a mental status examination. Which assessments would be most appropriate? A. Examining the patient's EEG B. Observing the patient as her or she takes an IQ test C. Observing the patient and inferring health or dysfunction D. Examining the patient's response to a specific set of questions

C

You are reviewing assessment data of a 45-year-old male patient and note pain of 8 on a scale of 10, labored breathing, and pale skin color on the electronic health record. This documentation is an example of: A. Hypothetical reasoning B. Diagnostic reasoning C. Data cluster D. Signs and symptoms

C

You are the triage nurse in the emergency department and perform the initial intake assessment on a patient who does not speak English. Based on your understanding of linguistic competence, which action would present as a barrier to effective communication? A. Maintaining a professional respectful demeanor B. Allowing for additional time to complete the process C. Providing the patient with a paper and pencil so he or she can write down the answers to the questions that you are going to ask D. Seeing if there are any family members present who may assist with the interview process

C

You examine the nail beds of a patient. Which finding indicates a normal angle? A. 60 degrees B. 100 degrees C. 160 degrees D. 180 degrees

C

You must be alert for which eye emergency symptoms? A. Floaters B. Epiphora C. Sudden onset of vision change D. Photophobia

C

Your are preparing the discharge plan for a patient with aphasia. What assessment should you include in the plan? A. Ask the patient to calculate serial 7s B. Ask the patient to name his or her grandchildren and their birthdays C. Ask the patient to demonstrate word comprehension by naming articles in the room or on the body as you point to them D. Ask the patient to interpret a proverb

C

Which cranial nerve is responsible for: Smell

CN I: Olfactory

Which cranial nerve is responsible for: Vision

CN II: Optic

Which cranial nerve is responsible for: Extraocular movement, pupil constriction, down and inward movement of the eye

CN III: Oculomotor

Which cranial nerve is responsible for: Down and inward movement of the eye

CN IV: Trochlear

Which cranial nerve is responsible for: Phonation, swallowing, tasting on the posterior third of the tongue

CN IX: Glossopharyngeal

Which cranial nerve is responsible for: Mastication and sensation of face, scalp, cornea

CN V: Trigeminal

Which cranial nerve is responsible for: Lateral movement of the eyes

CN VI: Abducens

Which cranial nerve is responsible for: Tasting on the anterior two thirds of the tongue, closing the eyes

CN VII: Facial

Which cranial nerve is responsible for: Hearing and equilibrium

CN VIII: Acoustic

Which cranial nerve is responsible for: Talking, swallowing, and sensory information from pharynx and carotid sinus

CN X: Vagus

Which cranial nerve is responsible for: Movement of trapezius and sternomastoid muscles

CN XI: Spinal

Which cranial nerve is responsible for: Movement of the tongue

CN XII: Hypoglossal

A patient is admitted to the emergency department after a motor vehicle accident. The trachea is deviated to the left side. This finding is characteristic of: A. Right-sided atelectasis. B. Right pleural adhesion. C. Aortic arch aneurysm. D. Right pneumothorax.

D

Moving the arm in a circle around the shoulder

Circumduction

A 46-year-old man requires an assessment of his sigmoid colon. Which instrument or technique is most appropriate for this examination? A. Rectal examination with an examining finger B. Proctoscope C. Ultrasound D. Colonoscope

D

A 54-year-old woman with 5 children has varicose veins of the lower extremities. Her most characteristic sign is: A. Reduced arterial circulation B. Blanching, deathlike appearance of the extremities on elevation C. Loss of hair on feet and toes D. Dilated, tortuous superficial bluish vessels

D

A caruncle is a(n) A. hard, painless nodule in the uterine wall. B. aberrant growth of endometrial tissue. C. vestibular gland located on either side of the vaginal orifice. D. small, red mass protruding from the urethral meatus.

D

A complete database is used to collect data rapidly and is A. often compiled concurrently with lifesaving measures. B. used to evaluate the cause or etiology of disease. C. used for a limited or short-term problem usually consisting of one problem, one cue complex, or one body system. D. used to perform a thorough or comprehensive health history and physical examination.

D

A full mental status examination should be completed if the patient A. develops dysphagia. B. has a new diagnosis of type 2 diabetes mellitus. C. complains of insomnia. D. has a change in behavior and the family is concerned.

D

A known risk factor for breast cancer is A. low breast tissue density. B. low-fat, low-cholesterol diet. C. breastfeeding an infant for more than 6 months. D. early menarche or late menopause.

D

A major characteristic of dementia is A. hallucinations. B. sudden onset of symptoms. C. cognitive deficits that are substance-induced. D. impaired short-term and long-term memory.

D

A nurse is evaluating the neurologic system of a patient. Which assessment would be included in the neurologic examination? A. Ask the patient if he or she has experienced nausea or vomiting. B. Check capillary refill. C. Check the patient for borborygmi. D. Observe the patient for ptosis.

D

A nurse is reviewing a patient's vital signs that have been taken by a nursing assistant and noted in the patient's medical record. The blood pressure measurement noted is 60/40. What should the nurse do based on reviewing this information in the patient's chart? A. Make a note to retake the blood pressure in an hour to see if there is a change. B. Review the information and as long as the nursing assistant has not reported any concern, proceed with other patient care. C. Have the nursing assistant retake the blood pressure because the reading is low. D. Go directly to the patient and retake the blood pressure.

D

A nursing assistant will be obtaining vital signs on a client who had a left-sided mastectomy two days prior. What specific instructions would the nurse provide to the nursing assistant in delegating this task? A. No special instructions are necessary B. Take blood pressure on the left arm C. Take blood pressure on both arms for a baseline D. Take blood pressure on the right arm

D

A patient complains that while studying for an examination he began to notice a severe headache in the frontotemporal area of his head that is throbbing and is somewhat relieved when he lies down. He tells the nurse that his mother also had these headaches. The nurse suspects that he may be suffering from: A. Hypertension. B. Cluster headaches. C. Tension headaches. D. Migraine headaches.

D

A patient is requesting pain medication and expresses a pain level of 9/10; however, the patient is up and smiling. How should you proceed? A. Complete a full physical examination B. Reposition the patient but withhold pain medication based on behavior C. Call the provider and suggest a substance abuse consult D. Use therapeutic communication techniques to determine the patient's pain scale goal and history of chronic pain

D

A patient is taking iron supplements. The patient should expect the stools to be: A. frothy B. clay-colored C. tarry and black D. nontarry and black

D

A patient seeks care for "debilitating headaches that cause excessive absences at work." On further exploration, the nurse asks, "What makes the headaches worse?" With this question, the nurse is seeking information about A. the patient's perception of pain. B. the nature or character of the headache. C. relieving factors. D. aggravating factors.

D

A patient tells the examiner that passing stools is painful. What term would the examiner use to document painful bowel movements? A. Flatulence B. Encopresis C. Occult D. Dyschezia

D

A positive Blumberg sign indicates: A. Possible aortic aneurysm B. Presence of renal artery stenosis C. Enlarge, nodular liver D. Peritoneal inflammation

D

A severe deficiency of thyroid hormone leading to nonpitting edema, coarse facial features, dry skin, and dry coarse hair is known as: A. Congenital hypothyroidism. B. Scleroderma. C. Hashimoto thyroiditis. D. Myxedema.

D

A woman at approximately 20 weeks' gestation reports lower right and/or left quadrant pain. Which of the following may be that cause of this pain? A. Appendicitis B. Constipation C. Urinary tract infection D. Stretching of the round ligament

D

A woman has come for an examination because of a missed menstrual period and a positive home pregnancy test. Examination reveals a cervix that appears cyanotic. This is referred to as the: A. Goodell sign B. Hegar sign C. Tanner sign D. Chadwick sign

D

A woman has come for health care reporting a thick white discharge with intense itching. These symptoms are suggestive of: A. Atrophic vaginitis B. Trichomoniasis C. Chlamydia D. Candidiasis

D

A woman has striae on the abdomen. Which color indicates long-standing striae? A. Pink B. Blue C. Purple-blue D. Silvery white

D

A woman who is pregnant for the first time is called a A. primipara. B. multipara. C. multigravida. D. primigravida.

D

Abdominal pain in the first trimester may be indicative of: A. Preterm labor B. Ectopic pregnancy C. Appendicitis D. All of the above

D

After examining a patient, you note: fever, increased respiratory rate, chest expansion decreased on left side, dull to percussion over left lower lobe, breath sounds louder with fine crackles over left lower lobe. These findings are consistent with: A. Bronchitis B. Asthma C. Pleural effusion D. Lobar pneumonia

D

Although a full mental status examination may not be required, you must be aware of the 4 main headings of the assessment while performing the interview and physical examination. These headings are: A. Mood, affect, consciousness, and orientation B. Memory, attention, thought content, and perceptions C. Language, orientation, attention, and abstract reasoning D. Appearance, behavior, cognition, and thought process

D

An abnormal sensation of burning or tingling is best described as: A. paralysis B. paraphasia C. paresis D. paresthesia

D

An adult patient's pulse is 46 beats per minute. The term used to describe this rate is A. weak and thready. B. tachycardia. C. sinus dysrhythmia. D. bradycardia.

D

An enlarged tongue (macroglossia) may accompany: A. cleft palate. B. hairy tongue. C. fissured tongue. D. Down syndrome.

D

An example of subjective data is A. decreased range of motion. B. crepitation in the left knee joint. C. arthritis. D. left knee has been swollen and hot for the past 3 days.

D

An older man asks if he is able to father children. In the aging male, when does infertility occur? A. At age 60, with the sudden decline in sperm production B. At approximately age 55 to 60, when testosterone levels are lower C. When the male is no longer able to achieve an erection D. There is no specific age; men may be fertile into their 80s and 90s

D

An ophthalmoscopic examination is an examination of the A. pharynx. B. nasal turbinates. C. inner ear. D. internal structures of the eye.

D

Anterior and posterior stability are provided to the knee joint by the: A. Medial and lateral menisci B. Patellar tendon and ligament C. Medial collateral ligament and quadriceps muscle D. Anterior and posterior cruciate ligaments

D

Any lump found in the breast should be referred for further evaluation. A benign lesion will usually have 3 of the following characteristics. Which one is characteristic of a malignant lesion? A. Soft B. Well-defined margins C. Freely movable D. Irregular shape

D

At the conclusion of the patient examination, the examiner should: A. Document findings after leaving the examining room B. Have findings confirmed by another practitioner C. Relate objective findings to the subjective findings for accuracy D. Summarize findings to the patient

D

Automatic associated movements of the body are under the control and regulation of: A. the thalamus B. the hypothalamus C. Wernicke's area D. the basal ganglia

D

Because of adolescents' developmental level, not all interviewing techniques can be used with them. Which techniques should be avoided? A. Facilitation and clarification B. Confrontation and explanation C. Empathy and interpretations D. Silence and reflection

D

Before examining a patient's ear with the otoscope, you would palpate which for tenderness? A. Helix, external auditory meatus, and lobule B. Mastoid, process, tympanic membrane, and malleus C. Pinna, pars flaccida, and antitragus D. Pinna, tragus, and mastoid process

D

The Landau reflex in the infant is seen when: A. The head is held and then flops forward as the baby is pulled to a sitting position by holding her or his wrists B. The infant's toes curl down tightly in response to touch on the ball of his or her foot C. The infant attempts to place his or her foot on the table while being held with the top of the foot touching the underside of the table D. The baby raises her or his head and arches the back, as in a swan dive

D

The components of a nail examination include A. clubbing, pitting, and grooving. B. shape, surface, and circulation. C. texture, toughness, and translucency. D. contour, consistency, and color.

D

The control of the body temperature is located in: A. Wernicke's area B. The thalamus C. The cerebellum D. The hypothalamus

D

The duct in the parotid gland that opens into the mouth opposite the second molar is A. the salivary duct. B. the Wharton's duct. C. the sublingual duct. D. the Stensen duct.

D

The examiner has estimated the jugular venous pressure. Identify the finding that is abnormal. A. Patient elevated to 30 degrees, internal jugular vein pulsation at 1 cm above sternal angle B. Patient elevated to 30 degrees, internal jugular vein pulsation at 2 cm above sternal angle C. Patient elevated to 40 degrees, internal jugular vein pulsation at 1 cm above sternal angle D. Patient elevated to 45 degrees, internal jugular vein pulsation at 4 cm above sternal angle

D

The examiner suspects a patient has coarctation of the aorta. Which assessment finding supports this suspicion? A. The thigh pressure is higher than in the arm B. The thigh pressure is equal to that in the arm C. The thigh pressure is unrelated to the arm pressure. There is no constant relationship; finding are highly individual D. The thigh pressure is lower than in the arm

D

The first heart sound (S1) is produced by the A. closure of the semilunar valves. B. opening of the semilunar valves. C. opening of the AV valves. D. closure of the AV valves.

D

The first step to cultural competency by nurse is to: a. Identify the meaning of health to the patient B. Understand how a health care delivery system works C. Develop a frame of reference to traditional health care practices D. Understand your own heritage and its bases in cultural values

D

The gradual loss of intra-alveolar septa and a decreased number of alveoli in the lungs of older adults cause A. spontaneous atelectasis. B. decreased dead space. C. hyperventilation. D. decreased surface area for gas exchange.

D

The hearing receptors are locate in which region? A. Vestibule B. Semicircular canals C. Middle ear D. Cochlea

D

The knee joint is the articulation of which three bones? A. Femur, fibula, and patella. B. Femur, radius, and olecranon process. C. Fibula, tibia, and patella. D. Femur, tibia, and patella.

D

The left upper quadrant (LUQ) contains the: A. Liver B. Appendix C. Left ovary D. Spleen

D

The most common sexually transmitted infection in the United States is A. trichomoniasis. B. syphilis. C. gonorrhea. D. chlamydia.

D

The most common site of cancerous breast tumors is in the A. upper inner quadrant. B. lower outer quadrant. C. lower inner quadrant. D. upper outer quadrant.

D

The most reliable indicator of pain in the adult is: A. The degree of physical functioning B. Nonverbal behaviors C. The MRI findings D. The patient's self-report

D

The nasal mucosa of an individual with rhinitis would be: A. moist and pink. B. swollen, boggy, and gray. C. pale with bright red bleeding. D. bright red and swollen.

D

The nurse administers an intravenous dose of pain medication. The nurse should reassess the patient in A. 30 minutes. B. 60 minutes. C. 5 minutes. D. 15 minutes.

D

The nurse is assessing muscle strength in a client and notes full range of motion against gravity with full resistance. The nurse would correctly document which of the following choices? A. Poor B. Fair C. Good D. Normal

D

The nurse is assessing the plantar reflex on a client and notes a fanning of the toes when the sole of the foot is stimulated. The nurse would correctly chart which of the following? A. Clonus B. Brudzinski sign C. Nuchal rigidity D. Babinski response

D

The nurse is completing a general survey assessing the level of consciousness of a person. Which of the following findings are expected in this assessment? A. No signs of acute distress are present B. Facial features symmetric with movement C. Patient appears drowsy and is having difficulty answering questions. D. Patient is alert and oriented to person, place, time, and situation.

D

The nurse is completing a general survey for an older adult and notices the patient demonstrates a wider gait with short, uneven steps. Which of the following would be the most important action of the nurse? A. Ask another nurse to assess the patient. B. Refer the patient to a geriatric health care specialist. C. Notify the physician immediately. D. Document this as normal findings.

D

The nurse is performing an assessment. Which of these findings would cause the greatest concern? A. Painful vesicle inside the cheek for 2 days B. Presence of moist, nontender Stensen's ducts C. Stippled gingival margins that snugly adhere to the teeth D. Ulceration on the side of the tongue with rolled edges

D

The nurse is performing range of motion of the cervical spine and asks the client to touch the chest with the chin. The nurse is assessing which of the following movements? A. Rotation B. Hyperextension C. Lateral flexion D. Flexion

D

The nurse is preparing to conduct a health history. Which of these statements best describes the purpose of a health history? A. To provide an opportunity for interaction between the patient and the nurse B. To provide a form for obtaining the patient's biographic information C. To document the normal and abnormal findings of a physical assessment D. To provide a database of subjective information about the patient's past and current health

D

The nurse uses health promotion activities with a new patient. What would this focus include? A. The nurse would try to change the patient's perception of disease B. The nurse would search for identification of biomedical model interventions C. The nurse would help to identify negative health act of the patient D. The nurse would empower the patient to choose a healthier lifestyle

D

The opening of an adult's parotid gland (Stensen's duct) is opposite to: A. Lower 2nd molar B. Lower incisors C. Upper incisors D. Upper 2nd molar

D

The relative proportion of glandular, fibrous, and adipose breast tissue depends on A. genetics. B. sex. C. environmental factors. D. nutritional state.

D

The reservoirs for storing milk in the breast are: A. Lobules B. Alveoli C. Montgomery glands D. Lactiferous sinuses

D

The second heart sound is the results of: A. Opening of the mitral and tricuspid valves B. Closing of the mitral and tricuspid valves C. Opening of the aortic and pulmonic valves D. Closing of the aortic and pulmonic valves

D

The student nurse demonstrates correct technique in using the stethoscope to auscultate heart sounds when he/she does which of the following? A. Performs assessment while the patient is watching television. B. Uses the bell to detect higher pitched sounds. C. Auscultates in only 4 locations. D. Makes sure earpieces fit snugly and are pointed to their nose.

D

The term "culturally competent" implies that the nurse: A. Is prepared in nursing B. Possess knowledge of the traditions of diverse peoples C. Applies underlying knowledge to providing nursing care D. Understands the cultural context of the patient's situation

D

The term rugae refers to A. an acute inflammation of the testes. B. a muscle that controls the size of the scrotum. C. a corpus spongiosum cone of erectile tissue. D. folds of thin skin of the scrotal wall.

D

The thickening and yellowing of the lens due to aging is described as: A. Presbyopia B. Floaters C. Macular degeneration D. Cataract

D

To assess a patient's abdomen by palpation, how should the nurse proceed? A. Avoid palpation of reported "tender" areas because this may cause the patient pain B. Quickly palpate a tender area to avoid any discomfort that the patient may experience C. Begin the assessment with deep palpation encouraging the patient to relax and take deep breaths D. Start with light palpation to detect surface characteristics and to accustom the patient to being touched

D

To screen for deep vein thrombosis, you would: A. measure the circumference of the ankle B. check the temperature with the palm of the hand C. compress the dorsalis pedis pulse, looking for blood return D. measure the widest point with a tape measure

D

Vaginal lubrication is provided during intercourse by: A. Labia minora B. Sebaceous follicles C. Skene glands D. Bartholin glands

D

What disease is characterized by a flat, expressionless, or masklike face; a staring gaze; oily skin; and elevated eyebrows? A. Cushing syndrome B. Scleroderma C. Acromegaly D. Parkinson disease

D

What facial bones articulate at a joint instead of a suture? A. Zygomatic B. Maxilla C. Nasal D. Mandible

D

What is a priority assessment for aging adults? A. Phobias B. General intelligence C. Irrational thinking patterns D. Sensory perceptive abilities

D

What is an advantage for using SBAR during staff communication? A. It avoids making recommendations. B. It provides a complete patient health history. C. It focuses on a comprehensive physical examination. D. It improves verbal communication and reduces medical errors.

D

What is the source of deep somatic pain? A. Pancreas B. Intestine C. Skin and subcutaneous tissues D. Bones and joints

D

What term refers to a linear skin lesion that runs along a nerve route? A. Shingles B. Dermatome C. Annular D. Zosteriform

D

When addressing a toddler during the interview, the health care provider should A. use detailed explanations. B. ask the child, before the caregivers, about symptoms. C. use nonverbal communication. D. use short, simple, concrete sentences.

D

When auscultating the heart of a newborn within 24 hours after birth, the examiner hears a continuous sound that mimics the sound of a machine. This finding most likely indicates A. a normal sound because of the thinner chest wall of the newborn. B. the presence of congenital heart disease. C. pathology only when accompanied by an increased heart rate. D. an expected sound caused by nonclosure of the ductus arteriosus.

D

When inspecting the ear canal of a patient, the examiner chooses which speculum for the otoscope? A. A short, broad one B. The narrowest for a child C. The longest for an adult D. The largest that will fit

D

When reading a medical record, you see the following notation: Patient states, "I have had a cold for about a week, and now I am having difficulty breathing." This is an example of: A. A past medical history B. A review of systems C. A functional assessment D. A reason for seeking care

D

When taking a health history from an adolescent, the interviewer should A. ask every youth about the use of condoms. B. have at least one parent present during the interview. C. ask about violence and abuse before asking about alcohol and drug use. D. interview the youth alone with a parent in the waiting area.

D

When testing for muscle strength, the examiner should: A. observe muscles for the degree of contraction when the individual lifts a heavy object. B. measure the degree of force that it takes to overcome joint flexion or extension. C. estimate the degree of flexion and extension in each joint. D. apply an opposing force when the individual puts a joint in flexion or extension.

D

Which category is appropriate in a cultural assessment? A. Family history B. Chief complaint C. Past medical history D. Health-related beliefs

D

Which finding in the prostate gland suggests prostate cancer? A. Symmetric smooth enlargement B. Extreme tenderness to palpation C. Boggy soft enlargement D. Diffuse hardness

D

Which finding is considered to be subjective? A. Temperature of 101.2F B. Pulse rate of 96 beats/min C. Measure weight loss of 20lbs since the previous measurement D. Pain lasting 2 hours

D

Which is an example of objective data? A. Patient's history of allergies B. Patient's use of medications at home C. Last menstrual period 1 month ago D. 2- X 5-cm scar present on the right lower forearm

D

Which is considered a common physiologic change that occurs with pain? A. Polyuria B. Hyperventilation C. Hyperactive bowel sounds D. Tachycardia

D

Which is least likely to indicate a possible malignancy? A. History of radiation therapy to the head, neck, or upper chest B. History of using chewing tobacco C. History of large alcohol consumption D. Marked tenderness

D

Which of the following assessments should be included as part of the body structure portion of the general survey? A. Sexual development, skin color, and overall appearance B. Gait and range of motion C. Facial expression, speech pattern, and dress D. Stature, nutrition, and symmetry

D

Which of the following behaviors demonstrated by an individual may be indicative of hearing loss? A. Not looking at the examiner when being questioned B. Talking in a high-pitched voice C. Speaking slowly with well-articulated consonants D. Frequently asking for the question to be repeated

D

Which of the following best illustrates an abnormality of thought process? A. Lability B. Compulsion C. Aphasia D. Blocking

D

Which of the following is an expected finding of assessment of the male genitourinary system? A. Right testes 1.5 cm, Left testes 3 cm. B. Patient complains of urinary frequency C. Scrotum enlarged, rugae flattened. D. Patient states that he urinates 4 to 5 times per day.

D

Which of the following is an expected response on the cover test? A. The uncovered eye is unable to maintain its gaze on a fixed object. B. The covered eye jumps to reestablish fixation when it is uncovered. C. The covered eye moves into a relaxed position. D. The covered eye maintains its position when uncovered.

D

Which of the following is considered when preparing to examine an older adult? A. Avoid physical touch to avoid making the older adult uncomfortable. B. Confusion is a normal, expected finding in an older adult. C. Be aware that loss will result in poor coping mechanisms. D. Base the pace of the examination on the patient's needs and abilities.

D

Which of the following is the most reliable assessment method to assess for developmental dysplasia of the hip in an infant? A. Allis test B. Gluteal fold comparison C. Tibial torsion D. Ortolani maneuver

D

Which of the following questions would the examiner ask to determine whether an individual has epistaxis? A. "Have you ever noticed any unusual lesions on the inside of your mouth?" B. "Do you have any difficulty with swallowing?" C. "Do you experience a runny nose frequently?" D. "Do you experience nosebleeds?"

D

Which of the following statements about mental status testing of children is correct? A. The behavioral checklist is useful to assess children who are 3 to 5 years old. B. Input from parents and caregivers is discouraged when assessing psychosocial development. C. The results of the Denver II screening test are valid for white, middle-class children only. D. Abnormal findings are usually r/t not achieving an expected developmental milestone.

D

Which of the following voice sounds would be a normal finding? A. Whispered "1-2-3" is audible and distinct. B. The voice transmission is distinct and sounds close to the ear. C. The whispered sound is transmitted clearly. D. The "eeeee" sound is clear and sounds like "eeeee."

D

Which of the social determinants of health has the greatest influence on a person's health? A. Work environment B. Neighborhood C. Education D. Poverty

D

Which statement best describes interpretation as a communication technique? A. Interpretation is the same as clarification B. Interpretation is a summary of a statement made by a patient C. Interpretation is used to focus on a particular aspect of what the patient has just said D. Interpretation is based on the interviewer's inference from the data that have been presented

D

Which statement best describes the purpose of a health history? A. To provide an opportunity for interaction between the patient and examiner B. To provide a form for obtaining the patient's biographic information C. To document the normal and abnormal findings of a physical exam D. To provide a database of subjective information about the patient's past and present health

D

Which statement is most appropriate to use when initiating an assessment of cultural beliefs with an older American Indian patient? A. "Are you of the Christian faith?" B. "Do you want to see a medicine man?" C. "How often do you seek help from medical providers?" D. "What cultural beliefs are important to you?

D

Which would be included in the database for a new patient admission to a surgical unit? A. All subjective and objective data gathered by a health practitioner from a patient B. All objective data obtained form a patient through inspection, percussion, palpation, and auscultation C. A summary of a patient's record, including laboratory studies D. All subjective and objective data, data gathered from a patient, and the results of any laboratory or diagnostic studies

D

While assessing a man for allergies, he states he is allergic to penicillin. Which response is best? A. "Are you allergic to any other drugs?" B. "how often have you received penicillin?" C. "I'll write your allergy in your chart so you won't receive any." D. "Please describe what happens to you when you take penicillin."

D

While caring from a preterm infant, you are aware that: A. Inhibitory neurotransmitters are in sufficient supply by 15 weeks gestation B. The fetus has less capacity to feel pain C. Repetitive blood draws have minimal long-term consequences D. The preterm infant is more sensitive to painful stimuli

D

While discussing the treatment plan, the nurse infers that the patient is uncomfortable asking the physician for a different treatment because of fear of the physician's reaction. In this situation, the nurse's verbal interpretation A. impedes further discussion. B. helps the nurse understand his or her own feelings in relation to the patient's verbal message. C. affects the nurse-physician relationship. D. helps the patient understand personal feelings in relation to his or her verbal message.

D

You are assessing short-term cognitive function. Which assessment shows the ability to lay down new memories? A. Noting whether the patient completes a thought without wandering B. A test of general knowledge C. A description of past medical history D. Use of the Four Unrelated Words Test

D

You are teaching a parent of an infant the health promotion activities to reduce the risk for acute otitis media. Which would you include in the teaching plan? A. Using pacifiers B. Increasing group daycare C. Avoiding breastfeeding D. Eliminating smoking in the house and car

D

You assess a patient who has 4+ edema of the right leg. What is the best way to document this finding? A. Mild pitting, no perceptible swelling of the leg B. Moderate pitting, indentation subsides rapidly C. Deep pitting, leg look swollen D. Very deep pitting, indentation lasts a long time

D

You have done some predischarge teaching with a new mother about the care of her infant. This teaching included instruction about findings in observing her infant she would need to report to the pediatrician that may indicate jaundice. Four days after discharge from the hospital, the mother calls the pediatrician to report something she has observed and about which she is concerned. You would evaluate that the mother best understood your teaching if she reported which of the following to the pediatrician. A. Purplish areas on the buttocks and sacral area B. Skin color changes when placed in side-lying position C. Irregular pink or red patches on the back of the neck D. Yellowing of skin and mucous membranes

D

You note a lesion during a skin assessment. Which is the best way to document this finding? A. Raised, irregular lesion the size of a quarter, located on dorsum of left hand B. Open lesion with no drainage or odor, approximately 1/4 inch in diameter C. Pedunculated lesion below left scapula with consistent red color and no drainage or odor D. Dark brown raised lesion, with irregular border, on dorsum of right foot, 3 cm in size, no drainage

D

Which lymph node is deep under the sternomastoid muscle?

Deep Cervical

Lowering a body part

Depression

Assessment information is derived from which components? A. Review of clinical record B. Interview and Health history C. Physical examination and Functional Assessment D. Cultural and spiritual assessment E. All of the above

E

Sexually transmitted infections place the pregnant women at risk for: A. Infertility B. Premature rupture of membranes C. Preterm labor D. Postpartum maternal infections E. All of the above

E

Women older than 35 years who desire a pregnancy are at possible risk for: A. Congenital defects B. Infertility C. Diabetes D. Hypertension E. All of the above

E

You are assessing an adolescent boy. The first physical sign of puberty is: A. Height spurt B. Penis lengthening C. Sperm production D. Pubic hair development E. Testes enlargement

E

Raising a body part

Elevation

What type of database is most appropriate when rapid collection of data is required and often compiled concurrently with lifesaving measures? A. Complete B. Focused C. Follow-up D. Emergency

Emergency

Moving the sole of the foot outward at the ankle

Eversion

Straightening a limb at a joint

Extension

True or False Gathering the history and completing the physical exam is mostly the responsibility of the physician. The nurse should gather the assessment tools and wait for the physician to complete the health assessment.

False

Bending a limb at a joint

Flexion

Moving the sole of the foot inward at the ankle

Inversion

Which lymph node is under the angle of the mandible?

Jugulodigastric (Tonsillar)

Which lymph node is at the base of the skull?

Occipital

Documentation of an eye examination can include the term PERRLA. What does this mean?

P: Pupils E: Equal R: Round R: Reactive (to) L: Light (and) A: Accommodation

Which lymph node is superficial to the mastoid process?

Posterior Auricular

Which lymph node is in the posterior triangle along the edge of the trapezius muscle?

Posterior Cervical

Which lymph node is in front of the ear?

Preauricular

Turning the forearm so that the palm is down

Pronation

Moving a body part forward and parallel to the ground

Protraction

Moving a body part backward and parallel to the ground

Retraction

Moving the head around a central axis

Rotation

Which lymph node is halfway between the angle and the tip of the mandible?

Submandibular

Which lymph node is behind the tip of the mandible?

Submental

Which lymph node is overlying the sternomastoid muscle?

Superficial Cervical

Turning the forearm so that the palm is up

Supination

Which lymph node is above and behind the clavicle?

Supraclavicular

Fill in the blanks. S1 is best heard at the _______ of the heart, whereas S2 is loudest at the _______ of the heart. S1 coincides with the pulse in the __________ and coincides with the ___ wave if the patient is on an ECG monitor.

apex base carotid artery R


Ensembles d'études connexes

Medical Terminology: Chapter Six Self-Test Questions

View Set

Environmental Science Chapter 4 Objectives

View Set

Question of the Day (From Core Concepts Anesthesia Review + Open Anesthesia) + Apex Review Questions

View Set

ExamCompass: Network+ Quiz - Network Utilities Command Line

View Set

Mr. McLin-English Literature and Composition Section 1-Credit recovery

View Set